Módulo 6 Med. Interna Flashcards

1
Q

A 50-year-old man with progressive renal failure over 4 years presents with recent onset of numbness of his feet and hands. He is on a dietary regimen of water and salt restriction. Examination shows decreased sensation to pinprick and vibration stimuli below the knee and in the hands, and absence of ankle reflexes. Blood studies show:

  • Hematocrit 35%
  • Sodium 140 mEq/L
  • Potassium 5.0 mEq/L
  • Urea nitrogen 98 mg/dL
  • Creatinine 8.5 mg/dL
  • Bicarbonate 20 mEq/L

Which of the following is the most appropriate next step in management?

(A) Treatment with vitamin B12
(B) Administration of bicarbonate supplements
(C) Arterial blood gas analysis
(D) Treatment with recombinant erythropoietin
(E) Initiation of dialysis
(F) Sural nerve biopsy

A

Respuesta: E

The correct answer is E. This patient with chronic renal failure is manifesting typical signs and symptoms of uremic peripheral neuropathy. Sensorimotor neuropathy in a “stocking and glove” distribution is the most common form. Usually, peripheral neuropathy develops when the glomerular filtration rate falls below 10% of normal values. This is one of the indications for starting renal replacement therapy (i.e., dialysis). Parenthetically, uremic manifestations that warrant initiation of dialysis treatment include pericarditis, coagulopathy, fluid overload not responsive to diuresis, hyperkalemia resistant to dietary restriction, severe acidosis (pH <7.20), and neurologic complications (e.g., encephalopathy, seizures, and neuropathy).

Peripheral neuropathy of renal failure is not due to vitamin B12 deficiency; thus, treatment with vitamin B12 (choice A) is not helpful in this case. Vitamin B12 deficiency may play a role in some cases of anemia associated with chronic renal failure.

Administration of bicarbonate supplements (choice B) may help in correcting acid-base disorders, but serum bicarbonate levels of 20 mEq/L are considered satisfactory and do not need any further intervention.

Arterial blood gas analysis (choice C) would be appropriate if metabolic acidosis were suspected.

Treatment with recombinant erythropoietin (choice D) is used to obviate insufficient production by the kidneys and correct anemia of renal failure. This treatment is started when the hematocrit falls below 30 to 35% in the absence of other causes of anemia (blood loss, vitamin B12 deficiency, or iron deficiency). In any case, it would not be beneficial for peripheral neuropathy.

Sural nerve biopsy (choice F) is rarely necessary in a clinical setting of chronic renal failure, since the underlying etiology is quite clear.

How well did you know this?
1
Not at all
2
3
4
5
Perfectly
2
Q

A 56-year-old woman is brought to the emergency department by her husband because she attempted suicide 2 hours ago by ingesting some unknown medication. Her husband says that she is taking medications for depression, anxiety, and hypertension, but he cannot name the drugs. He also says that his wife has “drinking problems.” The patient appears mildly confused. Her temperature is 38.3 C (101 F), blood pressure is 120/85 mm Hg, pulse is 130/min, and respirations are 22/min. ECG reveals prolonged QRS complexes. On examination, dilated pupils, flushed skin, and muscle twitching are noted. Hepatic transaminases are normal, and blood gas analysis shows a normal pH. Which of the following is the most likely cause of this patient’s symptoms?

(A) Alcohol
(B) Benzodiazepines
(C) Clonidine
(D) Monoamine oxidase (MAO) inhibitors
(E) Specific serotonin reuptake inhibitors (SSRIs)
(F) Tricyclic antidepressants

A

Respuesta: F

The correct answer is F. Tricyclic compounds (e.g., amitriptyline and imipramine) are among the drugs most commonly used by depressed patients in suicide attempts. Their toxic effects are mostly attributable to peripheral anticholinergic activity and “quinidinelike” action (sodium channel block) on the heart. Mild overdose is principally associated with anticholinergic effects, such as mydriasis, tachycardia, impaired sweating with flushed skin, dry mouth, constipation, and muscle twitching. More severe intoxication leads to cardiac arrhythmias, namely ventricular tachyarrhythmias and bradycardia. Prolongation of the QRS complex (> 0.1 sec) is typical and constitutes a more sensitive indicator of toxicity than serum drug levels. Seizures, severe hypotension, and coma are the most severe manifestations. Hyperthermia may result from impaired sweating and/or status epilepticus. Gastric lavage in the first hour following ingestion, supportive care, anticonvulsants, and appropriate antiarrhythmic drugs (e.g., sodium bicarbonate bolus, lidocaine, and phenytoin) are the recommended treatments.

Severe intoxication with alcohol (choice A) manifests with respiratory depression, hypothermia, and coma. Ethanol levels greater than 300 mg/dL produce coma in persons who are not chronic abusers, but regular drinkers can tolerate even higher levels.

Benzodiazepines (choice B), like alcohol, depress the activity of the cerebral cortex, cerebellum, and brainstem reticular activating system. Thus, acute benzodiazepine intoxication produces stupor, coma, and respiratory depression.

Clonidine (choice C) is an antihypertensive agent with sympatholytic properties. Thus, clonidine overdose results in bradycardia, hypotension, miosis, and respiratory depression. Similar signs are produced by ingestion of topical nasal decongestants.

Monoamine oxidase (MAO) inhibitors (choice D) represent a second-line treatment for major depression. Overdose induces ataxia, excitement, hypertension, and tachycardia. These toxic reactions can be triggered by concomitant ingestion of tyraminecontaining foods and beverages (aged cheese and red wine). Serotonin syndrome, characterized by fatal hyperthermia, may develop in patients on MAO inhibitors who take serotoninergic drugs, such as fluoxetine, meperidine, tryptophan, and dextromethorphan.

Specific serotonin reuptake inhibitors (SSRIs) (choice E), such as fluoxetine, are the “newer” generation of antidepressant drugs and are devoid of the anticholinergic effects of tricyclic compounds. Overdose with these drugs may, however, cause seizures.

How well did you know this?
1
Not at all
2
3
4
5
Perfectly
3
Q

An otherwise healthy 28-year-old man comes to the physician because of recent onset of chest pain. He started an active exercise program of weight lifting 1 week prior to the onset of symptoms. The pain began 24 hours ago and has been constant in intensity. It is sharp, localized to the precordial region, and exacerbated by movement. His temperature is 37 C (98.6 F), blood pressure is 124/78 mm Hg, pulse is 70/min, and respirations are 12/min. There is tenderness on palpation of the chest wall muscles in the precordial region. Which of the following is the most appropriate next step in management?

(A) Mental health screening tests
(B) Oral anti-inflammatory treatment and prevention of future muscle overuse
(C) Administration of an oral “GI cocktail” containing Xylocaine and antacid
(D) Performance of a Bernstein test
(E) ECG at rest and during exercise
(F) Chest x-ray examination
(G) Upper gastrointestinal endoscopy

A

Respuesta: B

The correct answer is B. Most cases of chest pain occurring in young patients are of noncardiac origin. In particular, chest pain arising in patients aged 18-45 is usually due to one of three causes: muscular chest pain, costochondritis (Tietze syndrome), or gastroesophageal reflux. Factors supporting a diagnosis of chest pain of muscular origin, in this case, include exacerbation with movement, tenderness on palpation, and onset following strenuous physical exercise. When the clinical picture is clear, no further diagnostic tests are necessary (or cost-effective). The patient should receive symptomatic treatment, which may include administration of anti-inflammatory drugs, rest, and appropriate instructions to avoid future overuse of chest wall muscles.

Mental health screening tests (choice A) are indicated when chest pain is suspected to be of psychological origin, usually anxietyrelated.

Administration of an oral “GI cocktail” containing Xylocaine and antacid (choice C) is an appropriate diagnostic measure when chest pain is thought to result from gastroesophageal reflux or spasm. Such cocktails usually contain a mixture of viscous Xylocaine and antacids in varying concentrations. Prompt relief of symptoms following ingestion supports a diagnosis of chest pain of esophageal origin.

Performance of a Bernstein test (choice D) is based on the administration of diluted HCl (0.1 N solution) into the esophagus. An esophageal origin of chest pain (reflux or spasm) is supported if this maneuver reproduces the patient’s symptoms.

An ECG at rest and during exercise (choice E) is not necessary in patients with a clear clinical picture of non-cardiac chest pain.

Chest x-ray examination (choice F) would be useless in this case. X-ray films are necessary when there are indications of pulmonary pathology, such as pneumonia or pleuritis.

Upper gastrointestinal endoscopy (choice G) is the most sensitive test for gastroesophageal reflux or peptic ulcer disease.

How well did you know this?
1
Not at all
2
3
4
5
Perfectly
4
Q

A 50-year-old man is evaluated in the emergency department following an automobile accident. An x-ray film of his leg fails to reveal an accident-related fracture, but does show a variety of abnormal features of the femur and tibia, including microfractures, increased bone density, cortical thickening, bowing, and overgrowth. Follow-up studies show marked cortical thickening of the bones of the head. Laboratory findings include normal serum calcium, normal serum phosphate, and elevated serum alkaline phosphatase with increased urinary excretion of pyridinoline cross links. Which of the following is the most likely diagnosis?

(A) Bone metastasis from prostate cancer
(B) Fibrous dysplasia
(C) Hyperparathyroidism
(D) Multiple myeloma
(E) Paget disease

A

Respuesta: E

The correct answer is E. This patient has Paget disease of bone, which is characterized by localized areas of hyperactive bone. The etiology is unknown. Most patients are older than 40, with a 3:2 male predominance. Histologic examination of affected bone shows both heavy osteoclastic and heavy osteoblastic activity, resulting in coarsely woven, thick trabeculae that are heavily calcified but structurally weak. The radiologic findings illustrated in the question stem are typical. Some patients, such as this one, are picked up incidentally when x-ray films are performed for other reasons. Other patients may come to medical attention because of symptoms related to peripheral nerves compressed at cranial ostia (including hearing loss and increasingly severe pain) or pathologic fractures, or because an alert physician notices bitemporal skull enlargement (frontal bossing), a hobbling gait, or bowing of the legs or thighs. Drug therapy (etidronate disodium and related compounds, calcitonin) is now available to influence calcium and phosphate metabolism and thus at least partially control the disease process.

Bone metastasis from prostate cancer (choice A) usually occur in men older than 60. There are typically fewer bony lesions unless very advanced, and it is usually associated with an elevated prostatespecific antigen (PSA).

Fibrous dysplasia (choice B) produces cystic bone lesions.

Hyperparathyroidism (choice C) may cause bone lesions similar to those of Paget disease, but would be accompanied by hypercalcemia.

Multiple myeloma (choice D) causes cystic bone lesions.

How well did you know this?
1
Not at all
2
3
4
5
Perfectly
5
Q

A 45-year-old man, who recently immigrated from Sudan, presents with painful urination and frequency for several months. He reports recurrent episodes of blood in the urine in the past year. Microhematuria is found on a urinary dipstick test. Blood tests reveal mild anemia. Which of the following would be most likely to yield the correct diagnosis?

(A) Examination of urine for eggs
(B) Ultrasound examination of the urinary tract
(C) Plain x-ray film of the lower abdomen
(D) Intravenous pyelography (IVP)
(E) Cystoscopy

A

Respuesta: A

The correct answer is A. These manifestations are highly suggestive of vesical schistosomiasis, which is endemic in the Middle East and many parts of Africa. It is due to Schistosoma haematobium, the adult form of which lives in the venules of the bladder. The eggs passed in the urine develop into the larval form, which infects snails. Infective larvae are excreted by the snails into water and penetrate through intact skin or mucous membranes into exposed persons. The larvae reach the portal circulation, where they mature, mate, and migrate to terminal venules. S. haematobium reaches the vesical plexus, whereas Schistosoma japonicum and Schistosoma mansoni reach the venules of the bowel, causing vesical and intestinal schistosomiasis, respectively. Cystitis caused by chronic schistosomiasis manifests with recurrent hematuria and predisposes to squamous cell carcinoma of the bladder. Search for ova in the urine is the fundamental diagnostic test.

Ultrasound examination of urinary tract (choice B) is the imaging method of choice when there are signs and symptoms of ureteral obstruction and hydronephrosis. Because nephrolithiasis is quite common, even in areas in which schistosomiasis is endemic, ultrasound is often performed first, but definitive diagnosis of schistosomiasis rests on finding the eggs in the urine.

A plain x-ray film of the lower abdomen (choice C) may show calcifications of the ureters and/or vesical wall.

Intravenous pyelography (IVP; choice D) may be useful in demonstrating blockage of the urinary tract but does not help in establishing a specific etiologic diagnosis.

Cystoscopy (choice E) may show alterations of the bladder mucosa, ulcers, and areas of squamous metaplasia in advanced disease.

How well did you know this?
1
Not at all
2
3
4
5
Perfectly
6
Q

A 33-year-old man has AIDS and a CD4 cell count of 180/mL. He has a history of Pneumocystis carinii pneumonia and Kaposi sarcoma of the small bowel. He is currently on combination therapy consisting of two nucleoside analogs (zidovudine and didanosine) and a protease inhibitor (saquinavir). He also receives prophylactic treatment with trimethoprim-sulfamethoxazole and an antimycobacterial medication. At present, his condition is stable. Which of the following vaccinations may be safely administered to this patient?

(A) Bacillus of Calmette-Guerin (BCG)
(B) Influenza
(C) Measles
(D) Mumps
(E) Oral polio vaccine
(F) Rubella
(G) Yellow fever

A

Respuesta: B

The correct answer is B. Influenza vaccination should be administered yearly to persons at increased risk of severe complications. Among such groups are AIDS patients, elderly persons, and patients with chronic pulmonary or cardiac diseases. Although recent studies have shown that HIV viremia may be transiently increased by influenza vaccination, this effect seems to have no clinical significance. Thus, influenza vaccination is currently recommended for AIDS patients. Immunosuppressed patients, however, may have a poor antibody response to vaccinations.

Vaccinations based on live attenuated viruses or microorganisms should not be used in AIDS patients or otherwise immunocompromised individuals since they can bring about serious postvaccinal infections. These vaccinations include BCG (choice A), measles (choice C), mumps (choice D), oral polio vaccine (choice E), rubella (choice F), and yellow fever (choice G).

How well did you know this?
1
Not at all
2
3
4
5
Perfectly
7
Q

A 78-year-old man is admitted to the hospital because of the acute onset of dysuria, frequency, profound malaise, and shaking chills. His temperature is 39.8 C (104 F), blood pressure is 105/62 mm Hg, pulse is 120/min, and respirations are 28/min. Examination reveals pronounced tenderness in the right costovertebral angle. Urinalysis shows:

  • Red blood cells 10/hpf
  • White blood cells 100/hpf
  • Protein 2+
  • Casts None

A urine sample is sent for cultures. Pending urine culture results, which of the following is the most appropriate next step in management?

(A) Single-dose administration of cephalexin
(B) Single-dose administration of trimethoprim-sulfamethoxazole
(C) Infusion of Ringer’s lactate solution
(D) Treatment with intramuscular ceftriaxone plus oral doxycycline
(E) Treatment with IV ampicillin and gentamicin

A

Respuesta: E

The correct **answer is E. **The clinical picture is consistent with acute pyelonephritis (upper urinary tract infection), a severe condition that should be promptly treated with wide-spectrum antibiotic therapy without waiting for the culture results to be available. Patients usually need to be admitted to the hospital. Blood and urine are cultured to identify the agent and determine susceptibility to antibiotics. Meanwhile, treatment with IV ampicillin and an aminoglycoside (usually gentamicin) is started. This combination affords a wide-spectrum coverage that is usually effective against the most common pathogens associated with urinary tract infections, in particular Escherichia and Proteus.

Single-dose administration of cephalexin (choice A) or trimethoprim-sulfamethoxazole (choice B) is used to treat uncomplicated episodes of cystitis in women. Men with cystitis usually have some underlying condition that needs to be investigated. Cystitis manifests with dysuria, frequency, and suprapubic discomfort, but the patient is generally afebrile.

Infusion of Ringer’s lactate solution (choice C), as well as any other supportive measure, although necessary, is not adequate treatment for such a severe infection.

Intramuscular ceftriaxone plus oral doxycycline (choice D) is used to treat sexually transmitted infections, such as acute epididymitis, in which both gonococcus and Chlamydia may be involved.

How well did you know this?
1
Not at all
2
3
4
5
Perfectly
8
Q

A 25-year-old woman presents with increasing weakness. She reports that she has had a history of menorrhagia and easy bruising. She has no other prior medical history. She has no family history of bleeding disorders and denies a history of substance abuse. On physical examination she is tachycardic and pale, with multiple bruises on her body. Laboratory studies show a low iron level and decreased mean corpuscular volume. Coagulation studies are notable for a normal prothrombin time (PT), a prolonged partial thromboplastin time (PTT), a normal platelet count, and a prolonged bleeding time. Which of the following is the most likely diagnosis?

(A) Aspirin ingestion
(B) Factor VII deficiency
(C) Factor IX deficiency
(D) Hemophilia A
(E) Von Willebrand disease (vWD)

A

Respuesta: E

The correct answer is E. Von Willebrand disease (vWD) is a genetic defect with variable transmission that results in the deficiency or derangement of the antigenic portion of factor VIII (von Willebrand factor). This antigen is involved in the intrinsic coagulation pathway and facilitates platelet-endothelium interaction. In vWD, there is prolongation of the PTT, which is indicative of a defect in the intrinsic pathway, and of the bleeding time, which is indicative of a defect in the factor VIII antigen and endothelium interaction. This patient is menorrhagic and iron deficient because of the bleeding diathesis.

Aspirin (choice A) permanently inhibits platelet adhesion by suppressing platelet prostaglandin synthesis, which results in defective platelet function. It should not cause an increase in prolonged partial thromboplastin time (PTT) since it does not affect the coagulation factors.

Factor VII (choice B) is in the extrinsic arm of the coagulation pathway, and deficiency of this factor should increase only the prothrombin time (PT).

Factor IX (choice C) causes hemophilia B, which affects males because of sex linkage of the genes. Isolated prolongation of PTT is expected in such a situation.

Similarly, hemophilia A (choice D) is due to a deficiency in factor VIII, not in the von Willebrand factor as seen in vWD. Hemophilia A affects males, given the X-linked recessive inheritance, and prolongs the PTT

How well did you know this?
1
Not at all
2
3
4
5
Perfectly
9
Q

A healthy 26-year-old man is brought to the emergency department after a motor vehicle accident in which he sustained an open fracture of his left tibia and fibula. Physical examination fails to disclose any signs of trauma to the head, neck, or abdomen. The next morning, the patient develops acute dyspnea and confusion. His temperature is 37.7 C (99.9 F), blood pressure is 105/65 mm Hg, and pulse is 100/min. There is no jugular venous distension. Scattered rales are heard bilaterally in multiple lung fields. He has a rapid and regular S2 and S3. A repeat chest x-ray film reveals diffuse pulmonary edema. Arterial blood gas determination (on room air) shows a pH of 7.52, a PCO2 of 29 mm Hg, and a Po2 of 70 mm Hg. Which of the following is the most likely diagnosis?

(A) Aspiration pneumonia
(B) Asthma
(C) Cardiac contusion
(D) Congestive heart failure
(E) Fat embolism

A

Respuesta: E

The correct answer is E. Fat emboli occur after the introduction of neutral fat into the venous circulation, occurring most commonly after bone trauma or fracture. The clinical scenario described is classic. After a latent period of 12-36 hours, during which the patient is asymptomatic, cardiopulmonary and neurologic deterioration occurs. Dyspnea, tachypnea, and tachycardia occur with radiographic findings of diffuse bilateral infiltrates, consistent with adult respiratory distress syndrome (ARDS). Reduction in arterial oxygen content is consistent with widespread lung injury. Treatment is supportive, and mortality is high.

Aspiration pneumonia (choice A) develops after the inhalation of virulent microbial flora or foreign bodies. It occurs more frequently in patients with an impaired level of consciousness (e.g., those using alcohol or drugs or those experiencing seizures) or in patients with swallowing or mechanical impediments (e.g., nasogastric tube). Pneumonia is characterized by the sudden onset of fever, purulent sputum, and cough. Aspiration of foreign bodies is usually through the right bronchus, with unilateral changes on chest x-ray. Aspirated organisms may originally produce an infiltrate on chest x-ray, but ultimately tissue necrosis and pulmonary cavitation may occur.

Asthma (choice B) is a disease of the airway characterized by increased responsiveness of the tracheobronchial tree to multiple stimuli. It is an episodic disease associated with dyspnea, cough, and wheezing. Acute exacerbation results in hypoxia, along with hypocapnia and respiratory alkalosis. The chest x-ray is usually normal, but may show hyperinflation.

Cardiac contusion (choice C) is the most common injury following blunt heart trauma. The major cause of contusion is direct impact force applied to the intact pericardium, usually occurring without fracture of the bony thorax. The contusion results in a discrete or disseminated hemorrhage of the myocardium. Damage may be confined to the myocardium, or be complicated by lacerations of the endo- or epicardial surfaces. Pericardial effusions occur in more than 50% of patients. A fibrinous reaction of the contusion site may cause pain and a friction rub. Pain is usually immediate, retrosternal, or anginal, simulating that of coronary thrombosis. It is responsive to oxygen. The patient may develop tachycardia and arrhythmias, with all types of ECG changes noted.

Congestive heart failure (choice D) occurs when the heart cannot provide sufficient output to satisfy the metabolic needs of the body. It is commonly termed “congestive” heart failure because of increased venous pressure (pulmonary congestion with left heart failure and peripheral edema with right heart failure). Failure is usually the result of chronic disease, including atherosclerotic coronary artery or valvular disease, or renal disease resulting in fluid imbalance. It is rare in healthy young adults, but may occur acutely following cardiac trauma. The most common symptom of left heart failure is shortness of breath. Though there are no ECG changes specific for heart failure, typical findings may reflect underlying disease. Chest x-ray films demonstrate cardiomegaly and pulmonary vascular congestion.

How well did you know this?
1
Not at all
2
3
4
5
Perfectly
10
Q

A 22-year-old woman with type 1 diabetes presents to the emergency department with anorexia, nausea, vomiting, and abdominal pain. She is recovering from pneumonia and has had much difficulty regulating her blood sugars lately. On arrival to the emergency department, her blood glucose is 760 mg/dL, sodium is 125 mEq/L, potassium is 3.0 mEq/L, bicarbonate is 12 mEq/L, chloride is 92 mEq/L, and her blood is positive for ketones by acetone screening. Which of the following is the most appropriate initial step in management?

(A) Broad coverage antibiotic therapy
(B) IV glucose and insulin
(C) IV hypertonic saline
(D) IV potassium
(E) IV saline

A

Respuesta: E

The correct answer is E. The management of diabetic ketoacidosis (DKA) requires a basic understanding of three concepts: the patient has some underlying trigger for the DKA, the patient is very volume depleted, and the patient has severe electrolyte abnormalities. The management therefore focuses on these issues. This patient has an anion gap acidosis (gap >20) and elevated blood glucose. The first step in the care of all DKA patients is prompt restoration of their volume status. This is the priority, as the ongoing diuresis from the elevated glucose will only worsen their acidosis.

Administration of broad coverage antibiotic therapy (choice A) would be appropriate if the patient has an underlying infection. However, this intervention has no place in the acute management of DKA.

Administration of IV glucose and insulin (choice B) is appropriate once the blood glucose falls below 250 mg/dL. It is important at all times to have insulin on board for diabetic patients. Once the glucose falls below 250 mg/dL during therapy, glucose must be given with the insulin to prevent hypoglycemia and assist with clearance of the ketone bodies.

Hypertonic saline should not be administered (choice C), since this patient’s corrected sodium is 133 mEq/L (any extra glucose in the sample of blood used to calculate serum electrolytes will decrease the measured serum sodium by 2.6 mEq/L per 100 mg/dL of glucose), which is acceptable.

Administration of IV potassium (choice D) will be needed as the patient’s acidosis begins to correct and serum potassium begins to decline. In DKA patients, total body potassium becomes depleted as a result of diuresis.

How well did you know this?
1
Not at all
2
3
4
5
Perfectly
11
Q

A 50-year-old man presents to his physician after an absence of 5 years. The physician notices that the man’s facial features have changed, with protrusion of the mandible and malocclusion of the teeth. On questioning, the patient reports that he has had to buy larger shoes and have his wedding ring enlarged because of trouble taking it off. Physical examination demonstrates fairly coarse body hair, prominent sweating, and irregularity of the surface of the nose and forehead related to enlarged sebaceous glands. The voice is deep and husky. This patient most likely has a tumor involving which of the following?

(A) Adrenal gland
(B) Parathyroid gland
(C) Pituitary gland
(D) Testes
(E) Thyroid gland

A

Respuesta: C

The correct answer is C. The features illustrated are typical of acromegaly, which is due to excess growth hormone produced by a pituitary adenoma. Other clinical features that may be seen include thickened, sometimes darkly pigmented skin, a barrel chest, tongue enlargement, and an increase in hat size. Joint symptoms, which may include a crippling degenerative arthritis, are frequent. Other problems include peripheral neuropathies due to nerve compression, headaches, visual changes (related to the pituitary tumor), cardiac disease, hypertension, and increased cancer risk. The diagnosis is usually made clinically and then substantiated with skull x-rays (showing cortical thickening and enlargement of the sella turcica) and plasma growth hormone levels.

Adrenal tumors (choice A) can cause hypo- or hypersecretion of mineralocorticoids, glucocorticoids, and epinephrine/norepinephrine.

Parathyroid tumors (choice B) can alter calcium metabolism.

Testicular tumors (choice D) can secrete androgens, estrogens, or other steroid hormones.

Thyroid tumors (choice E) can cause hypothyroidism; they rarely cause hyperthyroidism or disturbances in calcium metabolism (medullary carcinoma).

How well did you know this?
1
Not at all
2
3
4
5
Perfectly
12
Q

A 36-year-old woman presents with a 16-year-history of severe rheumatoid arthritis. She has had progressive difficulty walking for the past 3 months and is now virtually bedridden. Involved joints include proximal small joints of the hands, wrists, shoulders, knees, and ankles, with erosions and moderate deformities. Current medications include naproxen, methotrexate, and prednisone. Clinical examination shows mildly swollen and tender joints in the noted areas. Neurologic examination is notable for increased tone in both lower limbs, bilateral pathologically brisk reflexes in the upper and lower limbs, ankle clonus, positive Babinski sign, and minimal patchy sensory loss in the hands and feet. Grip strength is about 70% of normal. Which of the following is the most likely diagnosis?

(A) Atlantoaxial dislocation
(B) Cervical spondylotic myelopathy
(C) Osteoporotic spinal fractures and cord compression
(D) Peripheral neuropathy
(E) Spinal epidural lipomatosis

A

Respuesta: A

The correct answer is A. There is clear clinical evidence of corticospinal tract disease. Inflammation and destruction of the atlantoaxial joints is a crucial clinical diagnosis to make in the management of patients with severe rheumatoid arthritis. Local pain may be minimal, and the progression may be so slow that dysfunction and disability are attributed to peripheral arthritis. Episodes of deficit progression may occur and may be precipitated by unusual neck movements. Any kind of manipulation is absolutely contraindicated. Management requires urgent neuroradiologic (MRI) and neurosurgical evaluations.

Though cervical spondylotic myelopathy (choice B) is statistically the most common cause of cervical spinal cord compression in an adult, onset is typically after the age of 50. The clinical features may be indistinguishable, but radiologic examination is diagnostic. The atlantoaxial articulation is not a site for degenerative arthritis.

The patient is at risk for severe osteoporosis because of long-term use of prednisone. Osteoporotic spinal fractures (choice C) are associated with back pain and loss of vertebral height and predominantly involve the thoracic spine. Cervical level compression cannot occur in this situation.

There is no convincing clinical evidence of peripheral neuropathy (choice D). A mild glove-and-stocking sensory neuropathy is relatively common in rheumatoid arthritis; however, this neuropathy is usually benign and does not imply inflammation of the nerves. Patchy distinct sensory loss may also be due to spinal cord compression, carpal/tarsal tunnel syndromes, or mononeuritis multiplex. Reflexes are usually hypoactive or absent, rather than hyperactive.

Epidural fat (choice E), especially at the thoracic level, may increase sufficiently during chronic use of glucocorticoids to result in spinal cord compression. It should always be considered if bony compression is not documented and spinal cord symptoms remain unexplained. MRI is the diagnostic modality of choice. It is a rare condition.

How well did you know this?
1
Not at all
2
3
4
5
Perfectly
13
Q

A 70-year-old man with amyotrophic lateral sclerosis living in a nursing home is transported to the emer-gency department because of cough productive of foul-smelling sputum and breathing difficulties for 1 week. His temperature is 38.5 C (101.3 F), blood pressure is 140/84 mm Hg, pulse is 90/min, and respirations are 18/min. Inspection of his oral cavity reveals poor dental hygiene. A chest x-ray film shows a cavity with an air-fluid level within the left lower lobe in association with a mild pleural effusion. Which of the following is the most likely pathogen?

(A) Escherichia coli
(B) Klebsiella pneumoniae
(C) Legionella pneumophila
(D) Mixed anaerobic bacteria
(E) Pseudomonas aeruginosa
(F) Staphylococcus aureus
(G) Streptococcus pneumoniae

A

Respuesta: D

The correct answer is D. Cough productive of foul-smelling sputum should immediately suggest a pulmonary infection involving anaerobic bacteria. This is often associated with poor dental hygiene and/or conditions favoring aspiration, such as neurologic disorders, depressed level of consciousness, or tracheal/nasogastric tubes. Prevotella melaninogenica, Fusobacterium nucleatum, and anaerobic streptococci are among the most common pathogens. In this case, the patient has developed a pulmonary abscess, as the xray finding of a “cavity with air-fluid level” strongly suggests.

Escherichia coli (choice A) is a common cause of nosocomial pneumonia, defined as pneumonia occurring in hospitalized patients more than 48 hours after admission.

Klebsiella pneumoniae (choice B) is particularly associated with community-acquired pneumonia in patients with alcohol abuse or diabetes mellitus. It may also cause nosocomial (hospital-acquired) pneumonia.

Legionella pneumophila (choice C), one of the most common causes of community-acquired pneumonia, preferentially affects immunocompromised patients, heavy smokers, and patients with chronic obstructive pulmonary disease (COPD). Outbreaks of legionellosis result from exposure to contaminated sources, such as air conditioning towers or shower heads.

Pseudomonas aeruginosa (choice E) is frequently isolated in cases of nosocomial pneumonia, as well as in pneumonia occurring in cystic fibrosis patients.

Staphylococcus aureus (choice F) is one of the pathogens most frequently associated with nosocomial pneumonia.

Streptococcus pneumoniae (choice G) is the most common cause of community-acquired pneumonia. Pneumococcal pneumonia typically causes lobar consolidation and frequently follows an upper respiratory tract infection

How well did you know this?
1
Not at all
2
3
4
5
Perfectly
14
Q

A 71-year-old woman is admitted to the hospital for dehydration. She has a history of coronary artery disease, rheumatoid arthritis, hyperthyroidism, hypertension, and renal artery stenosis, but has been well over the past few months. She was brought to the hospital by her daughter, who found her weak and nauseated. The patient reports that, with the onset of summer, she had been feeling quite fatigued and has had significantly decreased oral intake and progressive nausea. Current medications include digoxin, atenolol, aspirin, acetaminophen, prednisone, fluoxetine, levothyroxine, and nifedipine. Admission laboratory tests show:

  • Sodium 132 mEq/L
  • Potassium 3.4 mEq/L
  • Bicarbonate 30 mEq/L
  • Chloride 98 mEq/L
  • Urea nitrogen 52 mg/dL
  • Creatinine 1.9 mg/dL
  • Leukocytes 5300/μm3
  • Hematocrit 48%

Her last set of laboratory data, obtained 7 months ago, revealed completely normal values. On the basis of these admission laboratory values, which of her medications requires dosing adjustment?

(A) Acetaminophen
(B) Atenolol
(C) Digoxin
(D) Fluoxetine
(E) Levothyroxine

A

Respuesta: C

The correct answer is C. All medications are cleared either by the liver, the kidney, or the lung. The majority are cleared by one of the first two routes, whereas inhalational anesthetics are largely cleared by the third. Many patients, especially inpatients on current day medical and surgical wards, have impaired renal function, liver function, or both, and very careful consideration to dosing adjustments must be made. In fact, most house-staff manuals have lists of drugs that commonly have altered dosing in renal failure. The importance of altering dosing regimens depends on both the drug in question and its therapeutic index. Some drugs have a very close overlap between therapeutic dosing and toxic dosing. Digoxin is a perfect example of a drug that highlights both of these points and is VERY commonly seen on inpatient drug lists. The difference between a therapeutic and toxic dose for this drug can result from an increase in creatinine of only 0.5 mg/dL or an increased dose of 125 μg per day. Since this patient has impaired renal function, as evidenced by the increased blood urea nitrogen and creatinine, the dose of digoxin should be lowered.

Acetaminophen (choice A) is an example of a drug that is cleared almost entirely by the liver and also has a very narrow therapeutic index. In cases of liver disease, acetaminophen toxicity is very common.

Atenolol (choice B) is a long-acting formulation of a beta-1 selective antagonist used for treatment of hypertension and ischemic heart disease. It is largely cleared by the liver.

Fluoxetine (choice D) is a selective serotonin uptake inhibitor that is used as an antidepressant medication. It is cleared primarily by the liver.

Levothyroxine (choice E) is a thyroid hormone that is cleared primarily by uptake to the liver and normal endocrine pathway utilization.

How well did you know this?
1
Not at all
2
3
4
5
Perfectly
15
Q

A 75-year-old man comes to the physician because of bleeding gums and increasing malaise for several months. He lives alone in poor economic conditions. Examination reveals numerous petechiae, which are mostly perifollicular on closer examination, on his legs. Several ecchymoses are also noted on the arms and legs, and splinter hemorrhages are seen in the nail beds. His hair is brittle. Laboratory studies show:

  • Hematocrit 30%
  • Mean corpuscular volume 90 mm3
  • Mean corpuscular hemoglobin 30 pg/cell
  • Prothrombin time (PT) 12 sec
  • Partial thromboplastin time (PTT) 80 sec

Which of the following is most likely deficient in this patient’s diet?

(A) Iron
(B) Vitamin A
(C) Vitamin B1 (thiamin)
(D) Vitamin B6 (pyridoxine)
(E) Vitamin B12 or folate
(F) Vitamin C (ascorbic acid)
(G) Vitamin K

A

Respuesta: F

The correct answer is F. The clinical picture is consistent with scurvy due to vitamin C deficiency. Nowadays, this is an infrequent disorder, but it may be occasionally seen in alcoholic patients or in elderly people who do not eat enough fresh vegetables and fruits. The most important known function of ascorbic acid is to act as an essential cofactor in the hydroxylation of collagen. Deficiency of ascorbic acid leads to defective collagen synthesis, which results in capillary fragility, petechiae, ecchymoses, poor wound healing, and abnormal hair. Small hemorrhages in a perifollicular distribution are highly characteristic of scurvy. Gingival bleeding is also frequent but does not occur in edentulous patients.

Iron deficiency (choice A) results in hypochromic microcytic anemia. Dietary deficiency is rare in industrialized countries. Iron deficiency is usually caused by chronic blood loss.

Vitamin A deficiency (choice B) causes night blindness and xerophthalmia. This dietary deficit is most frequent in patients with malabsorption who do not receive appropriate vitamin supplementation. It may develop in people who abuse mineral oil laxatives.

Vitamin B1 (thiamin) deficiency (choice C) is frequent in alcoholics and manifests with neurologic deficits, including paresthesias, footdrop, wristdrop, and absence of ankle and knee reflexes. In its most severe form, it will cause high-output cardiac failure and Wernicke-Korsakoff syndrome.

Vitamin B6 (pyridoxine) deficiency (choice D) is rare since this factor is present in virtually all foods. Subclinical forms, however, seem to be relatively frequent. Secondary deficiency due to drugs that act as pyridoxine antagonists (e.g., isoniazid, penicillamine, and estrogens) should be kept in mind. Overt pyridoxine deficiency manifests with seborrheic dermatitis, glossitis, cheilosis, peripheral neuropathy, and sometimes seizures.

Deficiency of vitamin B12 or folate (choice E) leads to similar hematologic consequences, namely megaloblastic anemia. However, neurologic deficits due to degeneration of the posterior and lateral columns of the spinal cord are associated with vitamin B12, but not with folate deficiency.

Vitamin K deficiency (choice G) can result in bleeding diathesis. This, however, is accompanied by an elevated prothrombin time caused by impaired synthesis of clotting factors VII, IX, X, and prothrombin

How well did you know this?
1
Not at all
2
3
4
5
Perfectly
16
Q

A 52-year-old woman with type 1 diabetes mellitus presents with increasingly severe right otalgia and foul purulent discharge from the ear canal. Examination reveals granulations in the right ear canal associated with extreme edema and erythema of the canal skin. The tympanic membrane cannot be visualized. Paresis of the right abducens nerve is also appreciated. A CT scan of the head demonstrates bone erosion around the ear canal, extending into the middle fossa. Which of the following is the most likely pathogen?

(A) Haemophilus influenzae
(B) Mixed anaerobic flora
(C) Pseudomonas aeruginosa
(D) Staphylococcus aureus
(E) Streptococcus pneumoniae
(F) Moraxella catarrhalis

A

Respuesta: C

The correct answer is C. Physical examination shows the features of external otitis, an infectious disease of the ear canal that is most commonly caused by gram-negative rods or fungi. In diabetic persons, this infection typically follows a particularly aggressive course and leads to osteomyelitis of the cranial base if not promptly treated. Cranial nerve palsies, especially involving the sixth nerve, may develop. CT scan in this case confirms the presence of bone involvement. This severe form, which is also known as malignant external otitis, is most often due to Pseudomonas aeruginosa. Ciprofloxacin is the treatment of choice for Pseudomonas infections.

Haemophilus influenzae (choice A), Streptococcus pneumoniae (choice E), and Moraxella catarrhalis (choice F) represent the most common etiologic agents of acute otitis media. This is often preceded by an upper respiratory viral infection that blocks the auditory tube, allowing accumulation of fluid and bacterial proliferation within the middle ear. Otalgia is severe, and the tympanic membrane appears opaque and bulging outward. Amoxicillin or erythromycin plus sulfonamide is the treatment of choice.

Mixed anaerobic flora (choice B) and Staphylococcus aureus (choice D), as well as Proteus and Pseudomonas, are the bacterial species most frequently involved in chronic otitis media. This manifests with chronic purulent discharge, little or no pain, and a perforated tympanic membrane. With time, conductive hearing loss develops unless appropriate medical and surgical therapy are established.

How well did you know this?
1
Not at all
2
3
4
5
Perfectly
17
Q

A 48-year-old man with a long history of smoking presents to the emergency department with a chief complaint of difficulty breathing for the past 2 days. His temperature is 38.3 C (101 F), blood pressure is 120/70 mm Hg, and pulse is 103/min. Dullness to percussion and decreased breath sounds are noted over the right lower lung field. An upright chest x-ray film reveals a significant right-sided pleural effusion. Decubitus films show layering of the effusion. A diagnostic thoracentesis is performed. The following results are obtained:

Pleural fluid:
- pH 7.18
- Glucose 40 mg/dL
- Protein 3.8 g/dL
- LDH 220 IU/L

Serum:
- Protein 7.0 g/dL
- LDH 320 IU/L

Which of the following is the most likely etiology of the effusion?

(A) An exudate of infectious etiology
(B) An exudate of malignant etiology
(C) A transudate of infectious etiology
(D) A transudate of noninfectious etiology

A

Respuesta: A

The correct answer is A. This is an exudative effusion by definition. The pleural fluid LDH to serum LDH ratio is <0.6, and the pleural fluid protein to serum protein ratio is >0.5, so choices C and D can be eliminated. Determining whether this is infectious or malignant is tricky. In malignant effusions (choice B), very rarely is the pleural fluid glucose level less than 60 mg/dL (15%). The subacute clinical scenario (2 days of symptoms) and the temperature to 38.5 C (101.3 F) generally indicate an infectious etiology. The low pH is seen in both complicated parapneumonic effusions and cancer and does not help in the diagnosis. Further testing, including Gram stain, cultures, and cytology, should be done on this fluid.

How well did you know this?
1
Not at all
2
3
4
5
Perfectly
18
Q

A 55-year-old man comes to medical attention because of progressive tightness of the skin over his hands and face. He also reports difficulties swallowing solid foods. Laboratory studies show elevated titers of antinuclear and anti-DNA-topoisomerase antibodies. Which of the following autoantibodies would be consistent with the CREST variant of this condition?

(A) Anti-centromere
(B) Anti-DNA topoisomerase 1
(C) Anti-double stranded DNA
(D) Anti-phospholipid
(E) Anti-ribonucleoprotein (anti-U1-RNP)
(F) Anti-Smith antigen

A

Respuesta: A

The correct answer is A. Skin changes, arthralgia, Raynaud phenomenon, and dysphagia are among the most common presenting symptoms of scleroderma (progressive systemic sclerosis, PSS). This immune-mediated fibrosing condition affects the skin, gastrointestinal tract, lungs, kidneys, and myocardium, leading to severe functional damage. The exact pathogenetic mechanism is unclear, but the fibrosing reaction is probably mediated by T-lymphocytes, with production of cytokines that stimulate fibroblastic growth and collagen synthesis. Anticentromere antibodies are specifically associated with the CREST variant of systemic sclerosis. This affects specific body sites and has a more favorable clinical course. Calcinosis, Raynaud phenomenon, esophageal dysmotility, sclerodactyly, and telangiectasia are the defining features.

Antinuclear antibodies (ANAs) of various types are found in PSS, but the most specific (although not highly sensitive) is anti-DNA topoisomerase 1, also known as SCL-70 (choice B). This is present in up to one third of patients with scleroderma.

Anti-double stranded DNA (choice C) and anti-Smith antigen (choice F) are ANAs specifically correlated with systemic lupus erythematosus.

Anti-phospholipid antibodies (choice D) may be found in association with other collagen vascular diseases, especially SLE, but may occur as an isolated manifestation (antiphospholipid antibody syndrome). It leads to recurrent arterial and venous thrombosis.

Anti-ribonucleoprotein (anti-U1-RNP) (choice E) is an antibody specifically correlated with a form of autoimmune disease encompassing features of SLE, scleroderma, and polymyositis, known as mixed connective tissue disease.

How well did you know this?
1
Not at all
2
3
4
5
Perfectly
19
Q

A 45-year-old woman presents to the emergency department complaining of acute abdominal pain. She has a history of a peptic ulcer for several years that has been treated with an H2 blocker. She denies diarrhea, nausea, or vomiting, and states that she does not use alcohol or nonsteroidal anti-inflammatory medications. The pain is constant and nonradiating. On examination, she is tachycardic, but does not have a fever. Abdominal examination is remarkable for rigidity and rebound tenderness. Rectal examination produces dark stool that is guaiac positive. Which of the following is the most appropriate next step in management?

(A) Abdominal CT scan
(B) Upright chest x-ray film
(C) Upper endoscopy
(D) Laparoscopic exploration
(E) Exploratory laparotomy

A

Respuesta: B

The correct answer is B. This patient most probably is suffering from a perforated ulcer and has free air in the peritoneum. Such patients often present with a rigid abdomen and rebound tenderness. The best way to detect this is to look for free air under the diaphragm, which is best achieved with an upright chest x-ray.

A CT scan may be useful if the upright film is negative and no diagnosis has been made yet (choice A).

Upper endoscopy may be needed if the diagnosis is not made with initial radiologic studies, but this is an emergency and laparotomy may be considered first (choice C).

Laparoscopic exploration might not be viable if the surgery needs to be done emergently (choice D).

Exploratory laparotomy (choice E) may be needed to correct the perforation, but the upright x-ray film should be obtained first to confirm the diagnosis.

How well did you know this?
1
Not at all
2
3
4
5
Perfectly
20
Q

A 56-year-old man presents with a 10-month history of increasing dyspnea on exertion and occasional dry cough. He has been smoking half a pack of cigarettes daily for 40 years and has a history of rheumatoid arthritis. Chest examination reveals mild hyperresonance in all lung fields and diminished breath sounds. Lung volume measurements show increased total lung capacity (TLC) and residual volume (RV), with an elevated RV:TLC ratio. Which of the following is the most likely diagnosis?

(A) Asthma
(B) Bronchiectasis
(C) Chronic bronchitis
(D) Emphysema
(E) Interstitial lung disease

A

Respuesta: D

The correct answer is D. Most cases of emphysema are associated with long exposure to cigarette smoking. Emphysema manifests with progressive respiratory difficulty and a characteristic increase in the anteroposterior diameter of the chest (barrel chest). Hyperresonance and diminished breath sounds are present on chest examination, whereas chest x-ray shows hyperinflation and frequent parenchymal bullae (especially subpleural). The most characteristic changes in pulmonary function tests include increased TLC and RV, with an elevated RV: TLC ratio, which indicates that there is a predominant expansion of RV at the expense of functioning lung parenchyma.

Asthma (choice A) is associated with a typical history of paroxysmal episodes of breathing difficulties with wheezing.

Bronchiectasis (choice B) refers to abnormal progressive enlargement of a bronchial segment, with resultant accumulation of secretions and recurrent bronchopneumonia. Bronchiectasis typically presents with cough productive of copious amounts of purulent, often foul-smelling sputum.

Chronic bronchitis (choice C) is clinically defined as productive cough occurring for at least 3 months in 2 or more consecutive years. This patient did not have a history of productive cough. Asthma, bronchiectasis, and chronic bronchitis are often associated with emphysema in a clinicopathologic picture referred to as chronic obstructive pulmonary disease (COPD).

Interstitial lung disease (choice E) is due to diffuse diseases of the interstitium, leading to decreased expansion of the lung parenchyma and, consequently, reduced TLC and RV. This picture is known as restrictive lung disease and is associated with sarcoidosis, pneumoconioses, idiopathic pulmonary fibrosis, and collagen vascular diseases (including rheumatoid arthritis).

How well did you know this?
1
Not at all
2
3
4
5
Perfectly
21
Q

A 62-year-old man presents with complaints of severe pain in his left wrist, which he says is episodic and has increased in frequency over the past year. He says that he cannot move the wrist when this happens. His father had similar problems before he died of kidney problems due to diabetes. The patient is also diabetic and is taking insulin. His physical examination is normal except for the limitation of motion of the left wrist. A CBC is normal, and serum chemistry findings are as follows:

  • Sodium 139 mEq/L
  • Potassium 3.9 mEq/L
  • Chloride 98 mEq/L
  • Calcium 9.2 mEq/L
  • Uric acid 4 mg/dL

Aspiration of the joint fluid shows rhomboidal positively birefringent crystals under polarized light. Which of the following is the most likely diagnosis in this patient?

(A) Degenerative joint disease
(B) Gout
(C) Pseudogout
(D) Rheumatoid arthritis
(E) Septic arthritis

A

Respuesta: C

The correct answer is C. The presence of rhomboidal, positively birefringent crystals in the joint aspirate is diagnostic of pseudogout. This type of crystal arthropathy usually occurs in persons older than 60 years. The crystals consist of calcium pyrophosphate and tend to deposit on the joint cartilage. Pseudogout is associated with a variety of metabolic disorders, such as diabetes, hypothyroidism, hyperparathyroidism, and Wilson disease. Management consists of NSAID administration for acute episodes. Colchicine can be used for prophylaxis.

Degenerative joint disease (choice A) tends to cause continuous, rather than episodic, pain, and there are no crystals in the joint fluid.

Gout (choice B) typically presents with pain in the first metatarsophalangeal joint or in the knee joint. The uric acid is increased, and the joint aspirate would show negatively birefringent needle-shaped crystals.

Rheumatoid arthritis (choice D) usually presents with pain in small joints and morning stiffness; it is associated with rheumatoid factor.

Septic arthritis (choice E) is associated with evidence of microorganisms in the joint aspirate or elsewhere in the body, and there are large numbers of neutrophils in the joint fluid.

22
Q

A 40-year old man complains of symptoms of an upper respiratory infection. He has a sore throat and reports cervical adenopathy. He has previously been healthy and has not taken any medications. Family history is negative. On physical examination, he has a lowgrade fever and a blood pressure of 105/70 mm Hg. He has left anterior cervical adenopathy, measuring 2.5’” × “’3 cm, that is nontender and immobile. No other adenopathy is palpable. The patient is sent home with instructions to take fluids and advised that symptoms should resolve within a week. He is reevaluated over 6 months for recurrent respiratory tract infections. Although the lymph node has regressed, it has not disappeared. The patient reports that it has waxed and waned in response to antibiotics. The patient undergoes a lymph node biopsy. Which of the following is the most likely diagnosis?

(A) Burkitt lymphoma
(B) Diffuse large cell lymphoma
(C) Follicular mixed lymphoma
(D) Follicular small cleaved cell lymphoma
(E) Immunoblastic lymphoma

A

Respuesta: D

The correct answer is D. Follicular small cleaved cell lymphoma is among the most common indolent non-Hodgkin lymphomas. It accounts for approximately 40% of all cases. Even without treatment, patients have waxing and waning lymphadenopathy. Over time, the disease will progress, necessitating chemotherapy. This is a low-grade malignant lymphoma.

Burkitt lymphoma (choice A) has been associated with the EpsteinBarr virus and is usually found in Africa. In the U.S., patients present with intra-abdominal tumors. In Africa, patients present with large extranodal tumors of the jaws and abdominal viscera.

Diffuse large cell lymphoma (choice B) is characterized by large malignant lymphocytes and is less common than follicular small cleaved lymphomas. It is among the more aggressive lymphomas.

Follicular mixed lymphomas (choice C) account for approximately 30% of patients. Follicular mixed lymphomas consist of large cell and small cleaved cell populations. This subtype is indolent, but is more aggressive than follicular small cleaved cell lymphoma.

Immunoblastic lymphoma (choice E) is a high-grade non-Hodgkin lymphoma and is rapidly fatal unless effective treatment is administered promptly.

23
Q

A 35-year-old man presents with a 5-year history of low back pain. The pain is severe enough to interfere with daily activities and is associated with prominent stiffness of the lower back. The pain lasts for 2-3 hours following awakening and persists (though in a less severe form) for most of the day. He has mild pain and swelling in both knees and was recently treated with local glucocorticoid injections for pain and tenderness in both heels. Two years ago, he had a self-limited episode of pain in the right eye associated with photophobia and visual blurring, which resolved spontaneously over 4 weeks. Physical examination is notable for bilateral tenderness over the sacroiliac joints, reduction in spinal flexion movements, and mild bilateral knee effusions. Cardiac auscultation is notable for a barely audible murmur, suggesting aortic regurgitation. The erythrocyte sedimentation rate (ESR) is 46 mm/hr, and a complete blood count (CBC) and biochemistry are normal. Which of the following is the most likely diagnosis?

(A) Ankylosing spondylitis
(B) Lumbar canal stenosis
(C) Lumbar degenerative arthritis
(D) Pseudogout
(E) Rheumatoid arthritis

A

Respuesta: A

The correct answer is A. This patient is exhibiting most of the typical features of ankylosing spondylitis, one of the members of the seronegative spondyloarthropathies. Other members of this group include the inflammatory arthritis and enthesitis (inflammation of the junction of ligaments and bone) associated with inflammatory bowel disease, psoriasis, and Reiter disease. Clinical features include male sex, young adult age group, sacroiliitis, largejoint arthritis, iritis, aortic regurgitation, and pain/tenderness at boneligament junctions. Low back pain and stiffness are usually the dominant symptoms. Treatment is symptomatic and usually involves nonsteroidal antiinflammatory agents.

Lumbar canal stenosis (choice B) is characterized by local or lumbosacral radicular pain that is induced and worsened by standing. Posture is crucial, as keeping the spine flexed relieves pain. For example, bicycling may be well tolerated, whereas walking is not. Ankylosing spondylitis may be a cause of this syndrome, although the most common cause is degenerative arthritis, typically in the setting of a congenitally narrow bony canal.

Trauma and degenerative disease of the lumbar spine (choice C) are the most common causes of low back pain. Severe and prolonged stiffness does not occur, nor does sacroiliitis.

Pseudogout (choice D) due to deposition of calcium pyrophosphate or hydroxyapatite comes in several clinical forms: acute inflammatory monarthritis, incidental radiologic abnormality, symmetric polyarthritis, and deforming-mutilating arthritis. Axial involvement is exceptional. Sacroiliitis does not occur, nor do the other inflammatory associations.

Rheumatoid arthritis (choice E) is a symmetric polyarthritis of the proximal small joints and large axial joints. Low back pain and sacroiliitis are not features. Appendicular involvement is limited to the midcervical spine or the atlantoaxial joints.

24
Q

A 35-year-old man comes to the emergency department because of the acute onset of colicky pain in his right upper abdominal quadrant and jaundice. He had been working as an engineer in a Middle East country for 2 years and returned to the U.S. 2 months ago. On admission, his temperature is 37.2 C (99 F). The liver is slightly tender and palpable 2 cm below the costal arch. A CT scan of the abdomen reveals a 15-cm cyst in the right hepatic lobe, compressing the common bile duct. The cyst contains smaller cysts, and its wall is focally calcified. Which of the following is the most likely diagnosis?

(A) Amebic abscess
(B) Bacterial abscess
(C) Carcinoma
(D) Echinococcosis
(E) Hemangioma

A

Respuesta: D

The correct answer is D. Echinococcosis or hydatid disease is caused by Echinococcus granulosus and is endemic in Mediterranean countries and throughout the Middle East. The sheep is the intermediate host, whereas the dog is the definitive host. Humans become infected by ingesting the eggs in dog feces. The eggs hatch in the duodenum and reach the liver, where they develop into hydatid cysts, which are found in the liver, but some may also be found in the lungs and rarely in other organs. After approximately 6 months, the primary cyst will produce daughter cysts, and the daughter cysts may generate third-generation cysts. The fluid in the cysts can produce anaphylactic shock if the cyst ruptures into a serosal cavity or the blood stream. Usually, hydatid cysts remain asymptomatic for long periods until they reach the size of a palpable mass in the liver, compress the bile ducts producing cholestatic jaundice, undergo secondary infection, or break into the biliary tree. Radiologically, the characteristic features include a large cyst enclosing smaller cysts and calcifications within the wall, allowing differentiation from abscesses, hemangioma, or cancer.

Amebic abscess (choice A) is a complication of Entamoeba histolytica infection, which is endemic in tropical and subtropical countries. The patient presents with a 1 to 2-week history of right upper quadrant pain, fever, and marked liver tenderness on palpation. CT scan allows differentiating this condition from Echinococcus cysts.

Bacterial abscess (choice B) is most commonly due to ascending cholangitis caused by a stone, stricture, or neoplasm. Insidious onset of right upper quadrant pain and fever are the presenting symptoms. CT and MRI are the diagnostic techniques of choice.

Carcinoma (choice C) of the liver is the most common cancer in African and Asian countries in association with hepatitis B. In the U.S., alcoholic and hepatitic cirrhosis are the most common predisposing conditions. CT and MRI studies are able to differentiate cancer from hepatic abscesses or cysts.

Hemangioma (choice E) is the most common benign hepatic neoplasm and often represents an incidental finding. MRI is most useful in establishing the diagnosis.

25
Q

A 55-year-old Caucasian man consults a physician because of weight loss. He has also had chronic epigastric pain, nausea, and diarrhea. On physical examination, the weight loss is evident and 3+ ankle edema is noted. A complete blood count is unremarkable. Clinical chemistry studies are notable only for a serum albumin of 1.5 g/dL. Endoscopy demonstrates marked thickening of gastric folds in the body of the stomach. Gastric biopsy demonstrates a markedly thickened mucosa, with atrophy of glands accompanied by marked foveal hyperplasia with mucous gland metaplasia in many areas. Which of the following is the most likely diagnosis?

(A) Deep gastritis
(B) Gastric lymphoma
(C) Linitis plastica
(D) Menetrier disease
(E) Pernicious anemia

A

Respuesta: D

The correct answer is D. This is Menetrier disease, an uncommon gastric disorder characterized by markedly thickened gastric folds. The disorder is idiopathic, with the gross and microscopic features illustrated in the question stem. Menetrier disease can cause decreased acid secretion (hypochlorhydria) and a protein-losing enteropathy with weight loss and edema secondary to decreased serum albumin that may clinically mimic cancer. Medical treatments are typically ineffective, and some patients require partial or complete gastric resection to control severe hypoalbuminemia.

Deep gastritis (choice A) is a common form of nonerosive gastritis, usually related to Helicobacter pylori infection.

Gastric lymphoma (choice B) can also produce markedly thickened rugae, but the microscopic description would mention large numbers of atypical lymphocytes.

Linitis plastica (choice C) is a term used for an aggressive form of adenocarcinoma in which individual abnormal mucous cells (sometimes with “signet-ring” morphology) penetrate the wall of the stomach and trigger a marked fibrotic reaction. It can also cause thickened rugae.

Pernicious anemia (choice E) is characterized by marked gastric mucosal atrophy, rather than thickening

26
Q

A 56-year-old woman undergoes neurologic evaluation for recent onset of seizures and a gradually worsening tingling sensation in her right arm. Examination reveals mild loss of strength in the right upper extremity. CT scan and MRI show a 5-cm dural-based mass in the left frontoparietal region. The mass is well demarcated and compresses the underlying parenchyma. T1 -weighted MRI scans after gadolinium enhancement show marked contrast enhancement within the lesion associated with a characteristic “dural tail.” There is prominent thickening of the overlying calvarial bone. Which of the following is the most likely diagnosis?

(A) Glioblastoma multiforme (GBM)
(B) Langerhans cell histiocytosis
(C) Meningioma
(D) Metastasis
(E) Paget disease
(F) Schwannoma
(G) Tuberculoma

A

Respuesta: C

The correct answer is C. Meningiomas are benign neoplasms of meningothelial origin. They grow from the dura (“dural-based”) toward the cerebral parenchyma, pushing without infiltrating the brain. This pushing pattern of growth, resulting in “welldemarcated” borders, is highly characteristic of any benign neoplasm. An additional MRI/CT feature pointing to meningioma is the presence of a “dural tail” following contrast enhancement. Meningioma is the most frequent of benign CNS tumors and is two times as common in women than in men. Surgical excision is usually easy when located in the cerebral convexities. Rarely, meningiomas may have malignant features, invade the brain, and even result in extracranial metastasis. Meningioma frequently expands into the skull, inducing reactive bone condensation, but this is not a sign of malignancy.

Glioblastoma multiforme (GBM) (choice A) is the most frequent of primary malignant tumors of the brain. It is an intraaxial tumor (i.e., grows within the brain) and thus can be easily ruled out in this case. GBM is at the most malignant end of the spectrum of astrocytomas, tumors of astrocytic origin.

Langerhans cell histiocytosis (choice B), in its most frequent monostotic form, produces a single osteolytic lesion within a bone, commonly in the calvarium.

A metastasis (choice D) may involve any brain structure, including dura mater, but most commonly metastatic disease manifests as multiple brain masses located preferentially at the gray-white matter junction.

Paget disease (choice E) is a common disease that produces marked thickening and irregularities of bones, especially the calvarium. Pain is a common symptom. It is not associated with an intracranial mass.

Schwannoma (choice F), like meningioma, is an extraaxial tumor. The great majority of schwannomas originate from the eighth cranial nerve in the cerebellopontine angle. Note that several synonyms are used to designate this tumor (acoustic neuroma or neurinoma is probably the most common).

Tuberculoma (choice G) is a large aggregate of granulomas due to mycobacterial infection. Rarely, a tuberculoma may develop in the dura mater and simulate a meningioma, but this is definitely an unlikely occurrence.

27
Q

A 36-year-old woman who has worked as a hospital nurse for the past 10 years comes to the physician because of fatigue, weight loss, cough with blood-tinged sputum, and night sweats for 4 months. She has had evening temperatures up to 38.5 C (101.3 F). Prior to this, she had been in good health. Chest examination is remarkable for bilateral rhonchi and coarse crackles in the upper lung fields. A chest x-ray film reveals bilateral pulmonary infiltrates and cavitary lesions in the upper lobes. Her records show that she became positive to the tuberculin skin test in her first year of work as a nurse but was not treated. Which of the following interventions at that time would most likely have prevented her current condition?

(A) Annual chest x-ray examination
(B) BCG vaccination
(C) Close observation only
(D) Isoniazid for 6 months
(E) Rifampin for 6 months

A

Respuesta: D

The correct answer is D. The patient is exhibiting typical symptomatology of active tuberculosis, which preferentially affects the posterior segments of the upper lobes. Isoniazid (300 mg/day) prevents active disease in most individuals who are recent “skin test converters,” defined as persons with a prior documented negative tuberculin test within 2 years of a newly positive test. Isoniazid prophylaxis is also recommended for all HIV-positive or immunocompromised patients and for close contacts of patients with active TB.

Annual chest x-ray examination (choice A) is not an adequate measure to prevent skin converters from developing active tuberculosis and exposes the patient to significant levels of radiation.

BCG vaccination (choice B) refers to immunization with live attenuated strains of mycobacteria. BCG vaccination is currently recommended in some countries for tuberculin-negative persons, including health professionals, who are repeatedly exposed to patients with tuberculosis.

Close observation (choice C) would be insufficient to prevent active tuberculosis in a skin test converter. It may be used in the follow-up of close contacts of patients with tuberculosis due to isoniazidresistant strains.

Rifampin for 6 months (choice E) is also indicated for close contacts of patients with isoniazid-resistant tuberculosis

28
Q

A 33-year-old man is brought to the local health clinic with an acute attack of what he calls “podagra.” He says that his father, who died at age 65 in a car accident, had the same disease. On examination, his right great toe is swollen, red, and extremely tender to touch. His temperature is 38.7 C (101.7 F). The patient is otherwise healthy and does not take any medications. Laboratory studies show leukocytosis and a serum uric acid level of 9.8 mg/dL. Which of the following is the most appropriate treatment at this time?

(A) Allopurinol and nonsteroidal anti-inflammatory drugs (NSAIDs)
(B) Aspirin
(C) Colchicine
(D) Intra-articular administration of corticosteroids
(E) NSAIDs alone
(F) Parenteral administration of corticosteroids

A

Respuesta: E

The correct answer is E. The patient’s self-diagnosis of gout is confirmed by his clinical signs and symptoms, family history, and elevated serum uric acid levels. Treatment of the acute attack of gout relies on NSAIDs. Indomethacin has been traditionally used for this purpose, but any of the NSAIDs are equally effective. The treatment should be continued until the symptoms have completely subsided (usually for 1 week).

It would be a mistake to begin treatment for acute gout and hyperuricemia at the same time with allopurinol and NSAIDs (choice A). If treatment for hyperuricemia is necessary, it should be started after all the acute symptoms have resolved. Rapid reduction of serum uric acid levels can precipitate or worsen acute attacks of gout. Allopurinol acts by inhibiting xanthine oxidase. It is usually administered at an initial dose of 100 mg/day and is increased until the urate level is within normal limits.

Aspirin (choice B) is contraindicated in gout because it increases serum urate concentration.

Colchicine (choice C) is effective but causes significant gastrointestinal side effects, such as abdominal cramps and diarrhea. It is therefore reserved for patients who should avoid NSAIDs because of allergies, peptic ulcer disease, or impaired renal function.

Intra-articular administration of corticosteroids (choice D) may be used to provide prompt relief in acute attacks of monoarticular gout. This treatment is reserved for patients unable to take NSAIDs.

The same is true for parenteral administration of corticosteroids (choice F), such as IV methylprednisolone, which is otherwise used for polyarticular gout

29
Q

A 25-year-old man develops malaise with high fevers and rigors, followed several hours later by abdominal pain, vomiting, jaundice, and itching. His girlfriend brings him to the emergency department. Physical examination is remarkable for right upper-quadrant tenderness and slight icterus. A urine sample is dark, and a fecal sample is pale. Liver function tests show marked elevations of serum bilirubin and alkaline phosphatase, as well as modest elevations of the transaminases. Blood cultures later show Escherichia coli. The patient undergoes emergency endoscopic sphincterotomy, which fails to improve drainage of the biliary tract. An endoscopic cholangiogram demonstrates many bulbous dilations of the intrahepatic bile ducts with a relatively normal appearing extrahepatic biliary tree. Which of the following is the most likely diagnosis?

(A) Caroli disease
(B) Crigler-Najjar syndrome
(C) Dubin-Johnson syndrome
(D) Gilbert syndrome
(E) Rotor syndrome

A

Respuesta: A

The correct answer is A. This is Caroli disease, complicated by ascending cholangitis. The question stem illustrates a typical presentation of this congenital cystic malformation syndrome of the intrahepatic biliary tree. The disease may also present in early childhood. Associated ascending cholangitis may be difficult to treat; it may also recur and cause hepatic abscesses. Caroli disease strongly predisposes for biliary cirrhosis, and about 10% of these patients develop cholangiocarcinoma (bile duct cancer). About half the patients with Caroli disease also have congenital hepatic fibrosis.

Crigler-Najjar syndrome (choice B) is an inherited enzymatic abnormality of the liver that causes serious unconjugated hyperbilirubinemia.

Dubin-Johnson syndrome (choice C) is an inherited enzymatic abnormality that causes a dark gray liver and conjugated hyperbilirubinemia.

Gilbert syndrome (choice D) is a mild inherited enzymatic disease with asymptomatic unconjugated hyperbilirubinemia.

Rotor syndrome (choice E) is similar to Dubin-Johnson syndrome, but without the liver discoloration.

30
Q

A 74-year-old woman suddenly became delirious 1 hour ago. She was well until 5 days ago, when she tripped and injured her right leg; she has been bedridden since the accident. On examination, her temperature is 37.2 C (99 F), blood pressure is 110/70 mm Hg, pulse is 110/min, and respirations are 32/min. Her heart, lungs, and abdomen are normal, and there are no focal motor or sensory deficits. Passive flexion of her right hip causes obvious pain. Pulse oximetry shows an oxygen saturation of 80%, and a chest x-ray is normal. An ECG shows sinus tachycardia. Which of the following is the most likely diagnosis?

(A) Acute cerebral hemorrhage
(B) Acute cerebral infarction
(C) Myocardial infarction
(D) Pulmonary infarction
(E) Pulmonary thromboembolism

A

Respuesta: E

The correct answer is E. This patient has classic risk factors for pulmonary thromboembolism (PTE): leg injury and bedridden state. Her tachycardia, tachypnea, decreased O2 saturation on pulse oximetry, and lowgrade fever are also consistent with the diagnosis. The chest x-ray is often normal in cases of PTE, as in this patient. Her presenting symptom, delirium, is most likely due to her hypoxic state.

Note that PTE is one of the diagnoses the physician is likely to miss if he or she does not actively think about it; this can be fatal to the patient. Shortness of breath is the most common symptom of PTE, and tachypnea is the most frequent sign. In young patients, the common signs and symptoms (cardiac and respiratory) of PTE may mimic anxiety, especially when other corroborating signs are absent. In older patients whose primary complaint is vague chest discomfort, the symptoms might be confused with those of myocardial infarction (MI) (choice C), causing the patient to be discharged without further pulmonary work-up after MI is ruled out. At the very least, always suspect PTE when a patient presents with tachycardia and tachypnea and consider ordering a ventilationperfusion (V/Q) scan if suspicions are high.

Acute cerebral hemorrhage and acute cerebral infarction (choices A and B) might also produce delirium, but one would expect other accompanying neurologic symptoms and signs as well. In addition, a cerebral insult would be less likely to produce this patient’s cardiopulmonary symptoms.

The diagnosis of MI (choice C) requires satisfying two of the following criteria: history of prolonged chest discomfort, evidence of ischemia or necrosis on ECG (e.g., ST or T wave changes, Q waves), or elevated cardiac enzymes.

Pulmonary infarction (choice D) usually signifies the presence of a small PTE and typically causes severe pleuritic pain.

31
Q

A 35-year-old man comes to the physician because of recurrent bouts of high fever with shaking chills for 1 week. He was on a safari trip in sub-Saharan Africa 1 month ago. Each attack begins with shaking chills lasting a few hours, followed by fever up to 41.0 C (106 F), which resolves with profuse sweating. These attacks have occurred every other day. The patient feels slightly tired between episodes of fever but otherwise is well. Examination reveals mild splenomegaly. The liver is not palpable. A blood smear at this time is negative for parasites. Which of the following is the most appropriate next step in diagnosis?

(A) Blood cultures
(B) Repeat blood smear every 8 hours for 3 days
(C) Repeat blood smear during an attack
(D) Repeat blood smear once more
(E) Start treatment with chloroquine

A

Respuesta: B

The correct answer is B. The recent travel history to a malariaendemic region, and especially the sequential occurrence of typical febrile attacks every other day, should alert the physician to the possibility of Plasmodium infection. This parasite should be looked for in peripheral blood smears. Thick blood films are used for detection, whereas thin blood films are used for identification of the Plasmodium species. The number of parasites varies considerably during the course of the disease, and Plasmodium falciparum (the most dangerous) is particularly difficult to detect. Thus, to establish the diagnosis, blood smears should be examined every 8 hours during and between febrile attacks for at least 3 days.

Blood cultures (choice A) are not useful in this case since Plasmodium parasites cannot be cultured. An ELISA test for P. falciparum has recently become available.

Repeating blood smears during an attack (choice C) or just once (choice D) may not be sufficient to demonstrate Plasmodium parasites in red blood cells.

Starting treatment with chloroquine (choice E) before establishing diagnosis is not appropriate, although chloroquine is the agent of choice for prophylaxis and therapy of malaria. However, chloroquine-resistant strains of P. falciparum are present throughout the world, including Africa. Patients with falciparum malaria should be hospitalized. In addition, before starting treatment for acute malaria, it is essential to establish whether the patient has received any other antimalarial medication in the previous days to avoid the risk of overdose.

32
Q

A 42-year-old alcoholic presents to the emergency department complaining of 12 hours of epigastric pain radiating to the back. One and a half hours ago he began to experience nausea and vomiting. His temperature is 38.4 C (101.2 F), pulse is 100/min, blood pressure is 110/70 mm Hg, and respirations are 18/min. Abdominal examination reveals mid-epigastric tenderness without guarding, rebound tenderness, or distension. Laboratory tests show a serum amylase of 1050 U/L, AST of 300 U/L, and a white blood cell count of 18,000/mm3 . A plain film of the abdomen shows several small bowel air-fluid levels. Which of the following is the most likely diagnosis?

(A) Acute cholecystitis
(B) Acute gastritis
(C) Acute pancreatitis
(D) Intestinal obstruction
(E) Perforated peptic ulcer

A

Respuesta: C

The correct answer is C. Acute pancreatitis is most often associated with gallstones and excessive alcohol ingestion. Patients usually present with epigastric pain that radiates to the back, nausea, and vomiting. Examination is significant for fever and tachycardia. Abdominal examination is variable. The diagnosis is established by demonstrating an increase in serum amylase. Serum lipase is also elevated. Leukocytosis and elevated liver enzymes are usually present.

Acute cholecystitis (choice A) usually begins with colicky pain that progresses and becomes generalized in the right upper quadrant. The classic triad of right upper quadrant pain, fever, and leukocytosis suggests the diagnosis. The patient usually presents with anorexia, nausea, and vomiting. Abdominal examination is significant for right upper quadrant rebound and guarding, as well as distention resulting from a paralytic ileus. Leukocytosis and liver enzyme abnormalities are usually present. The amylase level is normal.

Acute gastritis (choice B) is usually associated with illness or drugs. The presentation is variable, but patients generally complain of upper abdominal or epigastric pain, nausea, and vomiting. Physical examination is usually normal. Laboratory abnormalities are rare. Guaiac-positive stools may be present if hemorrhage occurs.

Intestinal obstruction (choice D) usually presents with colicky pain, abdominal distention, and bilious vomiting. Abdominal x-ray films demonstrate distended loops of bowel, air fluid levels, and a paucity of air in the distal colon.

Perforated peptic ulcer (choice E) presents with severe epigastric pain. Laboratory abnormalities are rare. The diagnosis is easily made by the presence of free intraperitoneal air on abdominal x-ray films

33
Q

A 50-year-old woman with a history of breast cancer treated with mastectomy develops multiple metastases. As her disease progresses, she also develops anemia accompanied by a decreased white blood cell count and platelet count. The possibility of a myelophthisic anemia is considered. Which of the following features on peripheral blood smear would most strongly support this diagnosis?

(A) Hypochromic macrocytes
(B) Nucleated red cells
(C) Ringed sideroblasts
(D) Schistocytes
(E) Sickled cells

A

Respuesta: D

The correct answer is D. Schistocytes are small, fragmented, red cells that are formed when red cells squeeze through spaces too small for them and are fragmented. This can happen in myelophthisic anemia, which occurs when tumor, fibrosis, or granulomatous disease obliterates large areas of the marrow cavity. Myelophthisic anemia can be very difficult to treat unless the underlying disease process is controlled, since the marrow simply does not have the necessary volume to produce enough blood cells. Some of these patients will have erythropoiesis in extra-marrow sites such as the spleen or liver. Patients with severe disease are often transfusion dependent.

Hypochromic macrocytes (choice A) are seen in folate deficiency anemia, vitamin B12 deficiency anemia, and the thalassemias.

Nucleated red cells (choice B) are immature forms that are seen as a nonspecific finding in many forms of anemia.

Ringed sideroblasts (choice C) are a feature of sideroblastic anemia, which is a problem with erythrocyte maturation and utilization of iron.

Sickled cells (choice E) are a feature of sickle cell anemia, and may superficially resemble schistocytes, but are of normal size.

34
Q

A 45-year-old woman comes to the emergency department because of the sudden onset of abdominal pain that radiates from the right hypochondrium to the shoulder. The pain is steady, with periodic exacerbations. The patient is afebrile. Ultrasound examination reveals a stone in the cystic duct. Which of the following is the most appropriate next step in management?

(A) Cheno- or ursodeoxycholic acid treatment
(B) Lithotripsy with bile salt treatment
(C) Endoscopic sphincterotomy with stone extraction
(D) Laparoscopic cholecystectomy
(E) Open cholecystectomy

A

Respuesta: D

The correct** answer is D**. This patient manifests the classic symptomatology of biliary colic, characterized by pain in the right hypochondrium, which often radiates to the right shoulder. Patients are most often anicteric and afebrile. Virtually all of these cases are due to a gallstone obstructing the cystic duct. If the stone is in the common bile duct, obstructive jaundice becomes a prominent sign. Laparoscopic cholecystectomy is the intervention of choice since the patients can be discharged within 48 hours after surgery, and the procedure is associated with minimal trauma to the abdominal wall.

Cheno- or ursodeoxycholic acid treatment (choice A) may help dissolve small cholesterol stones and represents an alternative therapy for asymptomatic patients or patients who decline surgery. This treatment is effective only when the gallbladder is functioning.

Endoscopic sphincterotomy with stone extraction (choice C) may be used for choledocholithiasis, especially in those patients who have already undergone cholecystectomy.

Lithotripsy with bile salt treatment (choice B) is also an alternative under similar circumstances.

Open cholecystectomy (choice E) is not the preferred treatment since it is associated with more severe abdominal trauma and longer convalescence compared with the laparoscopic method. However, if any complication should arise during a laparoscopic procedure, the surgeon may easily switch to a more conventional open cholecystectomy

35
Q

A 45-year-old white man presents to his physician because of difficulty with defecation and fresh blood on his stool. He has no other complaints. Vital signs and most of the physical examination are unremarkable. Rectal examination demonstrates a 4-cm papillary mass at the anal verge. No mass lesions in the liver or lungs are visible on CT. Wide local excision of the tumor demonstrates well-differentiated nonkeratinizing squamous carcinoma arising in condyloma acuminatum. This lesion is most closely associated with which of the following viruses?

(A) Herpes simple I
(B) Herpes simplex II
(C) HIV
(D) Human papilloma virus (HPV)
(E) Molluscum contagiosum

A

Respuesta: D

The correct answer is D. Approximately 3 to 5% of the distal large bowel cancers are epidermoid carcinoma, with either a nonkeratinizing squamous cell or a basaloid histologic pattern. They can arise in condyloma acuminata (genital warts), which are virally induced proliferations of warts due to infection by HPV. HPV can be transmitted with sexual contact. The more superficial of the epidermoid carcinomas can often be treated with wide excision alone. A combination of chemotherapy and radiation therapy is often successful for somewhat deeper lesions.

Herpes simplex I (choice A) and II (choice B) do not cause cancer.

HIV (choice C), when it produces clinical AIDS, may predispose to growth of condyloma, but is not itself directly carcinogenic, so HPV is a better answer.

The lesions of molluscum contagiosum (choice E) are volcano-like skin lesions that do not predispose for cancer. The disorder is caused by a poxvirus

36
Q

A 24-year-old man with no significant past medical history is brought to the emergency department (ED) by his friends after 2 days of unusual behavior. His friends report that the patient has made unwanted advances toward several women at work and that he seemed unconcerned when confronted by his boss. They also note that he had forgotten to attend two engagements in the past 2 days. The ED visit was prompted by a prolonged spell during which the patient seemed “in a daze,” picking at his clothes for several minutes while a friend was trying to talk to him. In the ED, he is awake and alert but slightly disoriented. There is no meningismus. He has poor short-term memory, but the neurologic examination is otherwise nonfocal. Vital signs are remarkable for a temperature of 38.1 C (100.5 F). A CT scan of the brain with and without contrast is read as normal. A lumbar puncture is performed with the following results:

  • Opening pressure 18 cm
  • Glucose 80 mg/dL
  • Protein 75 mg/dL
  • WBCs 25/mm3 with a lymphocytic predominance
  • RBCs 8/mm3

Which of the following is the most likely diagnosis?

(A) Bacterial meningitis
(B) Brain abscess
(C) Creutzfeldt-Jakob disease
(D) Hemorrhagic infarct
(E) Viral encephalitis

A

Respuesta: E

The correct answer is E. Herpes simplex virus can produce a fulminant encephalitis that has a tendency to begin in the inferior frontal and medial temporal lobes. This location often leads to behavioral abnormalities as the presenting symptoms, followed by seizures that may be complex partial, as in this case, or generalized tonic-clonic. Imaging may be initially normal, but MRI subsequently demonstrates inflammation in the medial temporal and inferior frontal lobes. The CSF profile shows a lymphocytic pleocytosis with some evidence of a hemorrhagic component. EEG may show periodic focal spikes on a background of slow or lowamplitude (“flattened”) activity.

Bacterial meningitis (choice A) typically causes meningismus and should show a neutrophil-predominant CSF profile with a decreased glucose.

Brain abscess (choice B) typically presents with high fever, headaches, focal neurologic complaints, and seizures. A head CT with contrast should demonstrate a ring-enhancing lesion with edema and mass effect.

Creutzfeldt-Jakob disease (choice C), a degenerative dementia caused by infective prion proteins, can present with behavioral symptoms but follows a more subacute course. The CSF is usually unremarkable.

Infarcts may rarely present with behavioral symptoms and no focal features, but a 2-day-old hemorrhagic infarct (choice D) would be apparent on a CT.

37
Q

A 55-year-old Caucasian man presents to a physician with complaints of fatigue and weakness. On physical examination, the man is pale, but no other abnormalities are noted. A complete blood count demonstrates a severe microcytic anemia. Stool is positive for occult blood. Esophagogastroduodenoscopy is negative. Colonoscopy demonstrates a large fungating mass of the cecum. CT scans of the chest and abdomen do not reveal metastatic disease. Prior to surgery, measurement of serum levels of which of the following substances would provide a baseline to assess the possibility of recurrent cancer later in life?

(A) Alpha fetoprotein (AFP)
(B) Carcinoembryonic antigen (CEA)
(C) Human chorionic gonadotropin (hCG)
(D) Prostate-specific antigen (PSA)
(E) Thyroid-stimulating hormone (TSH)

A

Respuesta: B

The correct answer is B. The patient has colon cancer. It is important before many cancer surgeries to measure serum levels of appropriate tumor markers, since elevation of these markers before surgery indicates that they can then be used for monitoring disease recurrence. In the case of colon cancer, appropriate markers include CEA, CA 19-9, and CA 125. CEA is actually a nonspecific marker for many solid tumors, but is particularly useful in colon cancer, where it is commonly elevated.

AFP (choice A) is a marker for liver, ovary, and testicular cancer.

hCG (choice C) is produced by many tumors but is not as good as CEA for colon cancer.

PSA (choice D) is a marker for prostate cancer, although it is increased in benign prostatic hyperplasia as well.

TSH (choice E) is produced by some choriocarcinomas

38
Q

A 20-year-old male college student comes to the physician because of dry cough, fever, headache, and muscle pains for 2 weeks. He reports that other residents of his fraternity house have recently developed similar symptoms. He denies the use of illicit drugs and is not homosexual. His temperature is 38.2 C (100.8 F), pulse is 90/min, and respirations are 18/min. On chest examination, no rhonchi or bronchial breathing is appreciated. A chest x-ray film shows multifocal interstitial opacities throughout the lung parenchyma. Blood studies show mild leukocytosis and a positive cold agglutinin test. Which of the following is the most likely pathogen?

(A) Influenza virus
(B) Mixed anaerobic bacteria
(C) Mycoplasma pneumoniae
(D) Pneumocystis carinii
(E) Streptococcus pneumoniae

A

Respuesta: C

The correct answer is C. This is the typical clinical presentation of primary atypical pneumonia, which is most frequently caused by Mycoplasma pneumoniae, and less frequently by viruses (influenza, respiratory syncytial virus, adenovirus, rhinoviruses, rubeola, and varicella virus), Chlamydia, or Coxiella burnetii. What differentiates primary atypical pneumonia from other forms of acute pneumonia is the predominantly interstitial, rather than intra-alveolar, inflammation and scarcity of “localizing” symptoms. M. pneumoniae infections are often associated with circulating cold agglutinins. Small outbreaks within close communities are characteristic of pneumonia due to this microorganism. Erythromycin is the drug of choice.

Influenza virus (choice A) causes infection in an epidemic pattern. The symptoms begin rapidly, with sore throat, nasal stuffiness, fever, chills, and muscle aches. Leukopenia is commonly seen. Superimposed bacterial pneumonia is frequent in chronically ill individuals and the elderly. In young people, influenza is usually a self-limiting disease that lasts approximately 1 week.

Mixed anaerobic bacteria (choice B) are responsible for cases of pneumonia associated with lung cavitation (abscess) and clinically manifesting with cough productive of foul-smelling sputum. There are usually predisposing conditions, such as poor dental hygiene or situations favoring aspiration.

Pneumocystis carinii (choice D) is a fungus that causes pneumonia in severely immunocompromised hosts, especially AIDS patients. The clinical picture is severe. Symptoms begin abruptly, with high fever, chills, malaise, and shortness of breath. Acute respiratory failure ensues if appropriate therapy (trimethoprimsulfamethoxazole) is not instituted.

Streptococcus pneumoniae (choice E) is the most common etiologic agent of community-acquired pneumonia, resulting in consolidation of a single lobe and high fever with rigors, productive cough, and profound malaise.

39
Q

A 53-year-old man with chronic alcoholism presents to a community clinic with complaints of fatigue and weakness. Screening blood chemistry studies demonstrate hypocalcemia and hypokalemia. Depletion of the body stores of which of the following minerals is also likely in this patient?

(A) Iodine
(B) Iron
(C) Magnesium
(D) Selenium
(E) Zinc

A

Respuesta: C

The correct answer is C. Magnesium deficiency usually results from inadequate intake coupled with defective renal or gut absorption. Chronic alcoholism is an important clinical cause of magnesium deficiency in the U.S., probably because of both inadequate intake and excessive renal excretion. Alcoholism may also affect calcium and potassium metabolism. Hypomagnesemia may also be seen in malabsorption, kwashiorkor, parathyroid disease (particularly after removal of a parathyroid tumor), and chronic diarrhea. Manifestations include anorexia, nausea, vomiting, lethargy, weakness, seizures, and tetany.

Iodine deficiency (choice A) can cause thyroid dysfunction with goiter.

Iron deficiency (choice B) can cause microcytic anemia, glossitis, and esophageal webs.

Selenium deficiency (choice D) is very rare but can cause cardiomyopathy.

Zinc deficiency (choice E) can cause growth retardation, night blindness, and skin and hair changes.

40
Q

A 50-year-old man presents with arthralgias, hepatomegaly, and increased skin pigmentation for 3 months. Examination reveals a bronze color of the skin. The liver is palpable 3 cm below the right costal arch, but there is no splenomegaly. Moderately advanced testicular atrophy is appreciated. An S3 sound is heard on cardiac auscultation. Laboratory studies show:

  • AST 80 U/L
  • ALT 70 U/L
  • Alkaline phosphatase 120 U/L
  • Bilirubin, total 1.5 mg/dL
  • Ferritin 400 ng/dL
  • Transferrin saturation 60%
  • Glucose, fasting 180 mg/dL

Serologic tests for hepatitis virus antibodies are negative. Which of the following investigations would be the most appropriate next step in diagnosis?

(A) CT or MRI studies of the liver
(B) Measurement of serum alpha 1-antitrypsin
(C) Measurement of urinary copper excretion
(D) Serum titers of antinuclear and antimitochondrial antibodies
(E) Determination of hepatic iron content in a liver biopsy

A

Respuesta: E

The correct answer is E. Hepatomegaly, arthralgias, and skin hyperpigmentation (which may give the characteristic “bronze” color) are among the most common initial signs of hemochromatosis. Testicular atrophy is often present as well. Hemochromatosis leads to iron overload affecting the whole organism, but the liver, heart, pancreas, and endocrine glands bear the most injurious effects of iron deposition. Thus, cardiomyopathy and diabetes frequently manifest because of myocardial and pancreatic damage. A gallop rhythm, with associated S3 sound, is evidence of cardiomegaly. Note the high ferritin levels and elevated transferrin saturation, indicating systemic iron overload. This disease is due to an autosomal recessive mutation of a gene (named HFE) encoding a protein that interacts with beta-2-microglobulin. The ensuing pathogenetic steps are still unclear. When hemochromatosis is suspected, the definitive diagnostic confirmation relies on determination of iron content in liver biopsies. Histologic evaluation of iron deposition is not sufficiently accurate.

CT or MRI studies of the liver (choice A) may demonstrate iron overload in the liver, but these radiologic methods are not sensitive.

Measurement of serum alpha-1-antitrypsin (choice B) is undertaken when alpha-1-antitrypsin deficiency is suspected. In this case, liver damage and other signs and symptoms indicate iron overload as the most likely underlying etiology.

Measurement of urinary copper excretion (choice C) is the initial diagnostic method of choice, together with ceruloplasmin serum levels, in the diagnosis of Wilson disease, because of excessive copper accumulation in liver and other organs.

Serum titers of antinuclear and antimitochondrial antibodies (choice D) are elevated in 90 to 95% of patients with primary biliary cirrhosis. These patients usually present with generalized pruritus, presumably secondary to retention of bile salts.

41
Q

A 23-year-old nun has a 6-month history of amenorrhea and galactorrhea. She is very concerned that others may believe that she is pregnant, and she vehemently denies such a possibility. Physical examination confirms that milk can indeed be expressed from both breasts, but it is otherwise unremarkable. The pelvic examination is also unremarkable, showing no uterine enlargement or ovarian masses. Visual fields are normal. A pregnancy test is negative. Once the diagnosis is confirmed, which of the following is the most appropriate management?

(A) Bilateral mastectomy
(B) Bromocriptine
(C) Streptozocin
(D) Systemic chemotherapy
(E) Tamoxifen

A

Respuesta: B

The correct answer is B. This is one of those “higher cognitive level” questions that bypasses the diagnosis and goes directly to management, on the logical assumption that unless you have made a tentative diagnosis, you cannot select the appropriate treatment. Here the clinical diagnosis is prolactinoma, from a pituitary microadenoma (too small to produce visual field defects). The first line of treatment is bromocriptine. Pituitary surgery can be performed if needed.

Bilateral mastectomy (choice A) would indeed wipe out milk production, but it would do so in a mutilating, totally unnecessarily radical way. The breasts are normal; all we need to do is stop their stimulation by prolactin.

Streptozocin (choice C) is the chemotherapeutic agent of choice when the islands of Langerhans have to be destroyed. Thus, it is indicated for inoperable tumors producing gastrin, insulin, or glucagon.

Choices D and E are offered for those who mistakenly assumed that abnormal milk production is a sign of advanced breast cancer. It is not.

42
Q

A 65-year-old man is brought to the emergency department 30 minutes after the acute onset of headache, nausea, and sudden loss of balance. The patient is unable to stand or walk, but there is no loss of muscular strength or sensory deficits. Consciousness is preserved, and the patient is oriented to person, space, and time. He says that he had hypertension for a long time but has not taken any antihypertensive therapy. His blood pressure is now 166/98 mm Hg, and his pulse is 65/min. A CT scan of the head reveals an acute intracranial hematoma. Which of the following is the most likely location of the hematoma?

(A) Cerebellum
(B) Cerebral white matter
(C) Epidural space
(D) Pons
(E) Putamen
(F) Thalamus

A

Respuesta: A

The correct answer is A. Among hypertensive intra-cerebral bleeds, cerebellar hematomas merit special consideration. A hematoma within a cerebellar hemisphere manifests with sudden onset of headache and ataxia in an otherwise lucid patient. This highly characteristic presentation warrants immediate medical attention because of the possibility of full recovery if properly treated.

An intracerebral hematoma in the cerebral white matter (choice B) is usually accompanied by lateralized motor and/or sensory deficits.

Epidural hemorrhage (choice C) is always of traumatic origin and manifests with the characteristic talk-and-die syndrome. There is a lucid interval between trauma and coma.

Following hemorrhages in the pons (choice D), the patient falls into immediate coma with quadriparesis and usually dies of cardiorespiratory arrest.

The putamen (choice E) is the most frequent site of hypertensive bleeding. Putamenal hemorrhage usually involves the adjacent internal capsule, leading to contralateral hemiparesis, hemianesthesia, and hemianopia.

Bleeding into the thalamus (choice F) may be suspected when the patient manifests hemianesthesia that precedes hemiparesis.

43
Q

A previously healthy 37-year-old woman is brought to the emergency department complaining of lethargy and fever. Her husband reports she has been increasingly somnolent. The patient is complaining of discomfort in her left thigh. She denies obvious trauma, although reports removing a splinter from her left calf 2 days ago while bringing firewood into the house. Her temperature is 40.0 C (104 F), blood pressure is 85/50 mm Hg, and pulse is 130/min. On examination, she has exquisite tenderness to palpation. There is mild edema and erythema over her left thigh. Laboratory studies show an elevated creatinine kinase, liver function tests, and white count. Bedside debridement reveals that soft tissue has started to necrose. One blood culture bottle turns positive 5 hours after being drawn. The cultures are positive for group A streptococci. The patient is given a diagnosis of necrotizing fasciitis. Which of the following is the appropriate management at this time?

(A) Administration of clindamycin
(B) Administration of erythromycin
(C) Administration of penicillin G
(D) Emergent surgery
(E) Initiation of vasopressor agents

A

Respuesta: D

The correct answer is D. With the progress of necrotizing fasciitis, the tenderness may evolve to anesthesia, as cutaneous nerves are infarcted. Surgery for debridement and fasciotomy is needed for diagnosis and treatment because the infection is extensive and rapidly progressive. Once the surgery is complete, antibiotics must be continued.

This patient has a shock-like syndrome that is the result of the pyrogenic exotoxin A produced by the bacteria. Clindamycin (choice A) arrests the production of the exotoxin. Surgery is of paramount importance, however.

Erythromycin (choice B) may be used if the patient had a penicillin allergy.

Penicillin G (choice C) is also adjunctive therapy because surgery is of paramount importance.

If the patient becomes progressively hypotensive and risks hemodynamic collapse, vasopressors may be needed (choice E). But, fluid resuscitation should be attempted first.

44
Q

A 45-year-old woman comes to the clinic for a newpatient visit. A physician last evaluated her 6 years ago. She denies chest pain, shortness of breath, or palpitations. She is not taking any medications and denies any prior medical problems. On physical examination, she is noted to have a lesion on her left earlobe. She first noticed the lesion 6 months ago, and mentions that it has doubled in size. It is painless. She works as a gardener and is exposed to the sun. She denies wearing a hat. The lesion is fixed, with an ulcerated center with glistening, heaped-up edges. Which of the following is the most likely diagnosis?

(A) Basal cell carcinoma
(B) Erythema nodosum
(C) Leukoplakia
(D) Melanoma
(E) Squamous cell carcinoma

A

Respuesta: A

The correct answer is A. Given this patient’s extensive sunlight exposure, she most likely has a basal cell carcinoma. This is the most common form of skin cancer. Risk factors include sun exposure and ultraviolet radiation. Excision may be needed to avoid metastasis.

Erythema nodosum (choice B) is a vascular disorder. Skin manifestations include erythematous and nodular lesions, typically on the anterior aspect of the tibia.

Leukoplakia (choice C) is caused by the Epstein-Barr virus. It is found on the lateral aspects of the tongue and is white in appearance.

Melanoma (choice D) has the maximum metastatic potential and may be nodular or radial. It may have irregular borders, may be of variegated coloration, be >4 mm in size, and be on any aspect of the body.

Squamous cell carcinoma (choice E) is the second most common form of skin cancer. They commonly occur on the lower lip.

45
Q

A 44-year-old man presents complaining of lethargy, morning weakness and somnolence, and an inability to concentrate at work. He notes that these symptoms have become progressively worse over the last 3 years. This started about 6 months after a devastating breakup with his fiancée, when he decided to begin binge eating. He denies any other medical problems. He no longer complains of depression and is involved in a positive relationship. He denies substance use. Vital signs are within normal limits. He is 65 inches tall and weighs 308 pounds, which represents a 118-lb weight gain over the last 3 years. The rest of the physical examination is unremarkable. Which of the following is the most appropriate next diagnostic step?

(A) Empiric trial of weight loss and exercise
(B) Laboratory tests for hypothyroidism
(C) Mask continuous positive airway pressure (CPAP)
(D) MRI of the head and neck for soft tissues
(E) Overnight oxygen saturation/apnea monitoring

A

Respuesta: E

The correct answer is E. This patient has the signs and symptoms of obstructive sleep apnea. This condition is characterized by poor nighttime sleeping resulting in daytime somnolence and the psychological consequences of prolonged sleep deprivation. Patients tend to be irritable, have difficulty concentrating on daytime tasks, and have an inappropriate lack of energy given their activity level. Although only 50% of obstructive sleep apnea patients are obese, obese patients tend to have an increased amount of redundant nasal and nasopharyngeal tissue, which makes their upper airway more likely to obstruct in the supine position. Diagnosis of obstructive sleep apnea is initially made by the appropriate history and symptoms and is confirmed by polysomnography, or sleep study, which documents apnea or hypopnea in addition to oxygen desaturations during these obstructive episodes.

Empiric trial of weight loss and exercise (choice A) is part of the treatment plan in patients who have obstructive sleep apnea, but it does not confirm the diagnosis. Also, anatomic abnormalities independent of the increase in soft tissues associated with obesity may be the source of the problem, which would not resolve with weight loss or exercise.

Laboratory tests for hypothyroidism (choice B) are not indicated at this time, given the lack of other symptoms consistent with a diagnosis of hypothyroidism.

Mask continuous positive airway pressure (CPAP) (choice C) should not be administered unless the patient has been evaluated clinically for the presence of obstructive sleep apnea, either with polysomnography or continuous oxygen saturation measurements. In addition, CPAP administration will not be funded without documented evidence of sleep apnea.

An MRI of the head and neck for soft tissues (choice D) is not indicated and may not provide any information regarding the pathology of the insomnia. In addition, obstructive sleep apnea is a dynamic process that would need to be viewed radiologically in real time. The presence of redundant soft tissues in the nasopharynx and oropharynx does not necessarily confirm the diagnosis of obstructive sleep apnea.

46
Q

A 30-year-old man is admitted to the hospital because of extensive burn injuries involving 45% of the total body surface. Twenty-four hours following admission, he develops oliguria. Laboratory studies show elevated BUN and serum creatinine, with a BUN:creatinine ratio of 30. Urine osmolality is 800 mOsmol/kg H2O, and fractional excretion of sodium is 0.5%.

(A) Aminoglycoside toxicity
(B) Analgesic toxicity
(C) Cardiogenic shock
(D) Goodpasture syndrome
(E) Henoch-Schönlein purpura
(F) Hyperuricemia
(G) Hypovolemic shock
(H) Malignant hypertension
(I) Multiple myeloma
(J) Myoglobinuria
(K) Postinfectious glomerulonephritis
(L) Prostatic hyperplasia
(M) Radiographic contrast toxicity
(N) Sickle cell disease
(O) Unilateral ureteral stone

A

Respuesta: G

The correct answer is G. In this case, oliguria and other signs of acute renal failure may result from either sustained hypotension (secondary to fluid loss) or acute tubular necrosis. The latter may result from hypoxic/ischemic damage to the tubular cells in the setting of shock or sepsis. Thus, prerenal azotemia is due to decreased renal perfusion, whereas renal azotemia is due to intrinsic damage to renal tubules. The treatment differs depending on the underlying cause. The parameters that allow the differential diagnosis between prerenal and renal azotemia are principally the fractional excretion of sodium (FRNa) and the ratio between BUN and serum creatinine. Renal failure due to hypovolemic shock, as in this case, will manifest with prerenal azotemia, associated with FRNa <1% and BUN:Cr ratio >20. Usually, urine is hyperosmolar compared with plasma.

Cardiogenic shock (choice C) may also give rise to acute renal failure because of prerenal azotemia (hypoperfusion). In this case, however, the most likely mechanism is hypovolemic shock secondary to loss of fluid.

However, acute renal failure secondary to radiographic contrast toxicity (choice M) develops within 24 h after administration of IV radiocontrast. Further, it is obvious that anuria in this case is due to obstruction, not to intrinsic renal damage.

Aminoglycoside toxicity (choice A) is a common cause of acute renal failure due to acute tubular necrosis, in which case FRNa is >1% since damaged tubular cells are unable to reabsorb sodium. In addition, the BUN:Cr ratio is <20, and urine osmolality approaches that of the plasma (250-300 mOsmol/kg).

Analgesic toxicity (choice B) is due to chronic ingestion of large amounts of such agents as NSAIDs, aspirin, and paracetamol. It may result in papillary necrosis and progressive renal interstitial damage. Such damage will eventually lead to chronic interstitial nephritis, manifesting with chronic renal failure. Polyuria due to inability of the kidneys to concentrate urine is a typical early sign.

Goodpasture syndrome (choice D) is a chronic inflammatory disorder affecting the lungs and kidneys. It is mediated by antibodies against the collagen of the basement membranes of the lungs and glomeruli. Hemoptysis and nephritic syndrome often progressing to renal failure are its clinical manifestations.

Henoch-Schönlein purpura (choice E) is a disease of children. Purpura, hematuria, abdominal pain, melena, and arthralgias constitute its clinical picture, and IgA-mediated vasculitis its pathologic substrate.

Hyperuricemia (choice F) may cause acute renal failure, which develops in patients with rapid cell turnover, namely patients with leukemia or lymphoma who are undergoing chemotherapy.

Malignant hypertension (choice H) may cause hematuria, proteinuria, and loss of renal function because of arteriolar damage. Blood pressure is very high, usually >220/120 mm Hg.

Multiple myeloma (choice I) is a plasma cell neoplasia associated with production of a monoclonal immunoglobulin. Fragments of this immunoglobulin (usually dimers of light chains) are filtered through the glomerulus and may precipitate in the tubules. This will result in proteinuria, hypertension, and progressive renal failure.

Myoglobinuria (choice J) may produce acute tubular necrosis in the setting of extensive crush injuries that cause necrosis of skeletal muscle.

Postinfectious glomerulonephritis (choice K) usually follows pharyngitis or impetigo due to group A Streptococcus and manifests with typical nephritic syndrome. Rarely, this condition may manifest with rapidly progressive glomerulonephritis (i.e., crescentic type), leading to acute RF.

Sickle cell disease (choice N) is one of the causes of renal papillary necrosis. Other causes include analgesic toxicity, obstructive uropathy with recurrent urinary tract infections, and diabetes.

Unilateral ureteral stone (choice O) would manifest with acute onset of colicky flank pain, associated with gross or microscopic hematuria. Acute renal failure does not ensue in this situation, unless the contralateral kidney is already impaired.

47
Q

A 70-year-old man is admitted to the hospital because of the acute onset of anuria, nausea, vomiting, and malaise. Recent medical history is remarkable for intravenous pyelography (IVP) performed 2 weeks ago and treatment with tricyclic antidepressants started 1 week ago. Insertion of a Foley catheter yields 700 mL of concentrated urine. Laboratory studies show high urine osmolality and low urine sodium.

(A) Aminoglycoside toxicity
(B) Analgesic toxicity
(C) Cardiogenic shock
(D) Goodpasture syndrome
(E) Henoch-Schönlein purpura
(F) Hyperuricemia
(G) Hypovolemic shock
(H) Malignant hypertension
(I) Multiple myeloma
(J) Myoglobinuria
(K) Postinfectious glomerulonephritis
(L) Prostatic hyperplasia
(M) Radiographic contrast toxicity
(N) Sickle cell disease
(O) Unilateral ureteral stone

A

Respuesta: L

The correct answer is L. The clinical picture is consistent with acute urinary retention secondary to prostatic hyperplasia, which is an example of postrenal azotemia. Acute onset of anuria is usually accompanied by signs of acute renal failure, i.e., nausea and vomiting, malaise, and obtunded sensorium. This acute clinical picture is usually preceded by a long history of urinary symptoms due to prostatic hyperplasia, namely progressively increasing hesitancy, decreased force of stream, and postvoid dribbling. Use of drugs with anticholinergic properties, such as tricyclic antidepressants, may precipitate acute urinary retention. The recent history of intravenous pyelography (IVP) should not mislead you. Radiologic contrast media may act as direct nephrotoxins.

Cardiogenic shock (choice C) may also give rise to acute renal failure because of prerenal azotemia (hypoperfusion). In this case, however, the most likely mechanism is hypovolemic shock secondary to loss of fluid.

However, acute renal failure secondary to radiographic contrast toxicity (choice M) develops within 24 h after administration of IV radiocontrast. Further, it is obvious that anuria in this case is due to obstruction, not to intrinsic renal damage.

Aminoglycoside toxicity (choice A) is a common cause of acute renal failure due to acute tubular necrosis, in which case FRNa is >1% since damaged tubular cells are unable to reabsorb sodium. In addition, the BUN:Cr ratio is <20, and urine osmolality approaches that of the plasma (250-300 mOsmol/kg).

Analgesic toxicity (choice B) is due to chronic ingestion of large amounts of such agents as NSAIDs, aspirin, and paracetamol. It may result in papillary necrosis and progressive renal interstitial damage. Such damage will eventually lead to chronic interstitial nephritis, manifesting with chronic renal failure. Polyuria due to inability of the kidneys to concentrate urine is a typical early sign.

Goodpasture syndrome (choice D) is a chronic inflammatory disorder affecting the lungs and kidneys. It is mediated by antibodies against the collagen of the basement membranes of the lungs and glomeruli. Hemoptysis and nephritic syndrome often progressing to renal failure are its clinical manifestations.

Henoch-Schönlein purpura (choice E) is a disease of children. Purpura, hematuria, abdominal pain, melena, and arthralgias constitute its clinical picture, and IgA-mediated vasculitis its pathologic substrate.

Hyperuricemia (choice F) may cause acute renal failure, which develops in patients with rapid cell turnover, namely patients with leukemia or lymphoma who are undergoing chemotherapy.

Malignant hypertension (choice H) may cause hematuria, proteinuria, and loss of renal function because of arteriolar damage. Blood pressure is very high, usually >220/120 mm Hg.

Multiple myeloma (choice I) is a plasma cell neoplasia associated with production of a monoclonal immunoglobulin. Fragments of this immunoglobulin (usually dimers of light chains) are filtered through the glomerulus and may precipitate in the tubules. This will result in proteinuria, hypertension, and progressive renal failure.

Myoglobinuria (choice J) may produce acute tubular necrosis in the setting of extensive crush injuries that cause necrosis of skeletal muscle.

Postinfectious glomerulonephritis (choice K) usually follows pharyngitis or impetigo due to group A Streptococcus and manifests with typical nephritic syndrome. Rarely, this condition may manifest with rapidly progressive glomerulonephritis (i.e., crescentic type), leading to acute RF.

Sickle cell disease (choice N) is one of the causes of renal papillary necrosis. Other causes include analgesic toxicity, obstructive uropathy with recurrent urinary tract infections, and diabetes.

Unilateral ureteral stone (choice O) would manifest with acute onset of colicky flank pain, associated with gross or microscopic hematuria. Acute renal failure does not ensue in this situation, unless the contralateral kidney is already impaired.

48
Q

A 52-year-old man presents with episodic precordial pain and dyspnea on exertion. His blood pressure is 110/85 mm Hg, pulse is 85/min, and respirations are 16/min. Examination reveals a harsh systolic murmur along the left sternal border radiating to the neck. ECG changes are consistent with left ventricular hypertrophy.

(A) Acute cholecystitis
(B) Acute pericarditis
(C) Acute pleuritis
(D) Angina pectoris
(E) Aortic dissection
(F) Aortic regurgitation
(G) Aortic stenosis
(H) Costochondritis
(I) Esophageal carcinoma
(J) Esophageal spasm
(K) Herpes zoster
(L) Mitral valve prolapse
(M) Myocardial infarction
(N) Myocarditis
(O) Pneumonia
(P) Prinzmetal angina
(Q) Psychological chest pain
(R) Pulmonary thromboembolism
(S) Reflux esophagitis
(T) Secondary pneumothorax
(U) Spontaneous pneumothorax
(V) Thoracic outlet syndrome

A

Respuesta: G

The correct answer is G. These clinical manifestations, especially the characteristics of the systolic murmur, are consistent with aortic stenosis, which is often associated with anginal pain. The patient also shows signs of impeding left ventricular failure, namely exertional dyspnea and left ventricular hypertrophy. Aortic stenosis manifesting in younger individuals (<50 years) is usually due to a congenitally abnormal aortic valve. In the elderly, calcification of aortic valve cusps is the most common cause.

Acute cholecystitis (choice A) manifests with abdominal pain in the right upper quadrant radiating to the back and right shoulder, fever, and leukocytosis. Occasionally, it may present with precordial pain.

Acute pericarditis (choice B) is associated with precordial pain, fever, and evidence of pericardial effusion, e.g., a pericardial rub. If pericardial effusion is abundant, cardiac tamponade may ensue.

Acute pleuritis (choice C) is characterized by lateralized chest pain that intensifies with deep breathing. Objective signs of pleural effusion, as well as a pleuritic rub, are usually present.

Angina pectoris (choice D), in its most common form, manifests with precordial pain that has a crushing or squeezing quality. The pain is triggered by exercise or emotional stress and relieved by rest or vasodilators.

Aortic dissection (choice E) is an emergency manifesting with excruciating chest pain radiating to the back, pulse and pressure discrepancies, onset of aortic regurgitation, and shock. A history of hypertension is often present.

Aortic regurgitation (choice F) is associated with a diastolic murmur and a wide differential between systolic and diastolic pressure.

Costochondritis (choice H), also known as Tietze syndrome, is due to inflammation of the chondrocostal junctions, which are swollen and sore.

Esophageal carcinoma (choice I) may produce chest pain but usually manifests with progressive dysphagia, anorexia, and weight loss. Alcohol and smoking habits are often present in the history.

Esophageal spasm (choice J) is a rare cause of chest pain, but it is usually associated with swallowing difficulties. An x-ray after barium may show evidence of spasm.

Herpes zoster (choice K) is characterized by a vesicular eruption along a dermatome, which may be followed by chronic burning pain, especially in elderly patients.

Mitral valve prolapse (choice L) is due to myxomatous degeneration of mitral valve leaflets. It results in a characteristic midsystolic click followed, in the more severe cases, by a regurgitation murmur.

Myocardial infarction (choice M) usually results in extremely severe chest pain radiating to the left shoulder. The pain typically lasts longer than 30 minutes and is not relieved by rest or vasodilators. Many exceptions exist to this classic presentation, such as painless infarction or pain with unusual localization or radiation.

Myocarditis (choice N) may be asymptomatic, or it may lead to the acute onset of arrhythmias and acute heart failure or the late onset of slowly progressive congestive heart failure.

Pneumonia (choice O) may cause chest pain along with cough, sputum production, and fever. Pulmonary infiltrates are present on chest x-ray.

Prinzmetal angina (choice P) characteristically manifests with precordial pain in the early morning and results from coronary vasospasm without atherosclerotic stenosis.

Psychological chest pain (choice Q) is commonly associated with underlying depressive, anxiety, or panic disorders.

Pulmonary thromboembolism (choice R) usually occurs in the setting of predisposing conditions and manifests with chest pain, respiratory distress, tachypnea, and tachycardia. These symptoms, however, may be mild and difficult to interpret. Chest x-ray films and ventilation-perfusion scans are mandatory in any patient with suspicious signs.

Secondary pneumothorax (choice T), in contrast to the spontaneous form, develops in lungs with preexisting alterations, most commonly in chronic obstructive pulmonary disease (COPD). COPD leads to formation of subpleural bullae that may rupture. Abrupt onset of pain and worsening of dyspnea develop.

Thoracic outlet syndrome (choice V) originates from anatomic or inflammatory alterations of the thoracic outlet (a cervical rib, for example), which may result in compression or irritation of the neurovascular bundle. Pain is triggered by movements of the upper extremity or shoulder and is associated with paresthesias and muscle weakness.

49
Q

A previously healthy 25-year-old man comes to the physician because of left chest pain that began suddenly 2 days ago at rest. He drinks alcohol on social occasions and does not smoke. His height is 186 cm (73 in), and his weight is 70 kg (154 lb). His temperature is 36.8 C (98.2 F), blood pressure is 120/80 mm Hg in both arms, pulse is 80/min and regular, and respirations are 22/min. Chest examination reveals decreased tactile fremitus, hyperresonance, and diminished breath sounds in the left hemithorax.

(A) Acute cholecystitis
(B) Acute pericarditis
(C) Acute pleuritis
(D) Angina pectoris
(E) Aortic dissection
(F) Aortic regurgitation
(G) Aortic stenosis
(H) Costochondritis
(I) Esophageal carcinoma
(J) Esophageal spasm
(K) Herpes zoster
(L) Mitral valve prolapse
(M) Myocardial infarction
(N) Myocarditis
(O) Pneumonia
(P) Prinzmetal angina
(Q) Psychological chest pain
(R) Pulmonary thromboembolism
(S) Reflux esophagitis
(T) Secondary pneumothorax
(U) Spontaneous pneumothorax
(V) Thoracic outlet syndrome

A

Respuesta: U

The correct answer is U. The clinical history and objective findings are highly characteristic of spontaneous (primary) pneumothorax, which often affects tall, thin men between 20 and 40 years of age. Chest pain and respiratory distress begin suddenly, usually at rest and often during sleep. Patients often seek medical attention days after the onset of symptoms. If the pneumothorax is large, diminished breath sounds, reduced tactile fremitus, and hyperresonance are present. The condition is thought to arise from spontaneous rupture of subpleural bullae in otherwise normal lungs.

Acute cholecystitis (choice A) manifests with abdominal pain in the right upper quadrant radiating to the back and right shoulder, fever, and leukocytosis. Occasionally, it may present with precordial pain.

Acute pericarditis (choice B) is associated with precordial pain, fever, and evidence of pericardial effusion, e.g., a pericardial rub. If pericardial effusion is abundant, cardiac tamponade may ensue.

Acute pleuritis (choice C) is characterized by lateralized chest pain that intensifies with deep breathing. Objective signs of pleural effusion, as well as a pleuritic rub, are usually present.

Angina pectoris (choice D), in its most common form, manifests with precordial pain that has a crushing or squeezing quality. The pain is triggered by exercise or emotional stress and relieved by rest or vasodilators.

Aortic dissection (choice E) is an emergency manifesting with excruciating chest pain radiating to the back, pulse and pressure discrepancies, onset of aortic regurgitation, and shock. A history of hypertension is often present.

Aortic regurgitation (choice F) is associated with a diastolic murmur and a wide differential between systolic and diastolic pressure.

Costochondritis (choice H), also known as Tietze syndrome, is due to inflammation of the chondrocostal junctions, which are swollen and sore.

Esophageal carcinoma (choice I) may produce chest pain but usually manifests with progressive dysphagia, anorexia, and weight loss. Alcohol and smoking habits are often present in the history.

Esophageal spasm (choice J) is a rare cause of chest pain, but it is usually associated with swallowing difficulties. An x-ray after barium may show evidence of spasm.

Herpes zoster (choice K) is characterized by a vesicular eruption along a dermatome, which may be followed by chronic burning pain, especially in elderly patients.

Mitral valve prolapse (choice L) is due to myxomatous degeneration of mitral valve leaflets. It results in a characteristic midsystolic click followed, in the more severe cases, by a regurgitation murmur.

Myocardial infarction (choice M) usually results in extremely severe chest pain radiating to the left shoulder. The pain typically lasts longer than 30 minutes and is not relieved by rest or vasodilators. Many exceptions exist to this classic presentation, such as painless infarction or pain with unusual localization or radiation.

Myocarditis (choice N) may be asymptomatic, or it may lead to the acute onset of arrhythmias and acute heart failure or the late onset of slowly progressive congestive heart failure.

Pneumonia (choice O) may cause chest pain along with cough, sputum production, and fever. Pulmonary infiltrates are present on chest x-ray.

Prinzmetal angina (choice P) characteristically manifests with precordial pain in the early morning and results from coronary vasospasm without atherosclerotic stenosis.

Psychological chest pain (choice Q) is commonly associated with underlying depressive, anxiety, or panic disorders.

Pulmonary thromboembolism (choice R) usually occurs in the setting of predisposing conditions and manifests with chest pain, respiratory distress, tachypnea, and tachycardia. These symptoms, however, may be mild and difficult to interpret. Chest x-ray films and ventilation-perfusion scans are mandatory in any patient with suspicious signs.

Secondary pneumothorax (choice T), in contrast to the spontaneous form, develops in lungs with preexisting alterations, most commonly in chronic obstructive pulmonary disease (COPD). COPD leads to formation of subpleural bullae that may rupture. Abrupt onset of pain and worsening of dyspnea develop.

Thoracic outlet syndrome (choice V) originates from anatomic or inflammatory alterations of the thoracic outlet (a cervical rib, for example), which may result in compression or irritation of the neurovascular bundle. Pain is triggered by movements of the upper extremity or shoulder and is associated with paresthesias and muscle weakness.

50
Q

A 54-year-old woman comes to the physician because of recurrent substernal pain that manifests after meals and on reclining. She reports transient relief from taking antacids or drinking milk. Examination is otherwise unremarkable, but the patient is extremely anxious, fearing that she may have heart problems like her father, who died at age 84 of a “heart attack.”

(A) Acute cholecystitis
(B) Acute pericarditis
(C) Acute pleuritis
(D) Angina pectoris
(E) Aortic dissection
(F) Aortic regurgitation
(G) Aortic stenosis
(H) Costochondritis
(I) Esophageal carcinoma
(J) Esophageal spasm
(K) Herpes zoster
(L) Mitral valve prolapse
(M) Myocardial infarction
(N) Myocarditis
(O) Pneumonia
(P) Prinzmetal angina
(Q) Psychological chest pain
(R) Pulmonary thromboembolism
(S) Reflux esophagitis
(T) Secondary pneumothorax
(U) Spontaneous pneumothorax
(V) Thoracic outlet syndrome

A

Respuesta: S

The correct answer is S. Despite the vague familial history of heart problems, the clinical symptomatology is classic for gastroesophageal reflux or reflux esophagitis. The onset of pain in the recumbent position and soon after meals, along with the relief provided by antacids or milk, is virtually diagnostic. Upper endoscopy is the diagnostic procedure of choice to establish a definitive diagnosis and to obtain biopsy specimens of the esophageal mucosa.

Acute cholecystitis (choice A) manifests with abdominal pain in the right upper quadrant radiating to the back and right shoulder, fever, and leukocytosis. Occasionally, it may present with precordial pain.

Acute pericarditis (choice B) is associated with precordial pain, fever, and evidence of pericardial effusion, e.g., a pericardial rub. If pericardial effusion is abundant, cardiac tamponade may ensue.

Acute pleuritis (choice C) is characterized by lateralized chest pain that intensifies with deep breathing. Objective signs of pleural effusion, as well as a pleuritic rub, are usually present.

Angina pectoris (choice D), in its most common form, manifests with precordial pain that has a crushing or squeezing quality. The pain is triggered by exercise or emotional stress and relieved by rest or vasodilators.

Aortic dissection (choice E) is an emergency manifesting with excruciating chest pain radiating to the back, pulse and pressure discrepancies, onset of aortic regurgitation, and shock. A history of hypertension is often present.

Aortic regurgitation (choice F) is associated with a diastolic murmur and a wide differential between systolic and diastolic pressure.

Costochondritis (choice H), also known as Tietze syndrome, is due to inflammation of the chondrocostal junctions, which are swollen and sore.

Esophageal carcinoma (choice I) may produce chest pain but usually manifests with progressive dysphagia, anorexia, and weight loss. Alcohol and smoking habits are often present in the history.

Esophageal spasm (choice J) is a rare cause of chest pain, but it is usually associated with swallowing difficulties. An x-ray after barium may show evidence of spasm.

Herpes zoster (choice K) is characterized by a vesicular eruption along a dermatome, which may be followed by chronic burning pain, especially in elderly patients.

Mitral valve prolapse (choice L) is due to myxomatous degeneration of mitral valve leaflets. It results in a characteristic midsystolic click followed, in the more severe cases, by a regurgitation murmur.

Myocardial infarction (choice M) usually results in extremely severe chest pain radiating to the left shoulder. The pain typically lasts longer than 30 minutes and is not relieved by rest or vasodilators. Many exceptions exist to this classic presentation, such as painless infarction or pain with unusual localization or radiation.

Myocarditis (choice N) may be asymptomatic, or it may lead to the acute onset of arrhythmias and acute heart failure or the late onset of slowly progressive congestive heart failure.

Pneumonia (choice O) may cause chest pain along with cough, sputum production, and fever. Pulmonary infiltrates are present on chest x-ray.

Prinzmetal angina (choice P) characteristically manifests with precordial pain in the early morning and results from coronary vasospasm without atherosclerotic stenosis.

Psychological chest pain (choice Q) is commonly associated with underlying depressive, anxiety, or panic disorders.

Pulmonary thromboembolism (choice R) usually occurs in the setting of predisposing conditions and manifests with chest pain, respiratory distress, tachypnea, and tachycardia. These symptoms, however, may be mild and difficult to interpret. Chest x-ray films and ventilation-perfusion scans are mandatory in any patient with suspicious signs.

Secondary pneumothorax (choice T), in contrast to the spontaneous form, develops in lungs with preexisting alterations, most commonly in chronic obstructive pulmonary disease (COPD). COPD leads to formation of subpleural bullae that may rupture. Abrupt onset of pain and worsening of dyspnea develop.

Thoracic outlet syndrome (choice V) originates from anatomic or inflammatory alterations of the thoracic outlet (a cervical rib, for example), which may result in compression or irritation of the neurovascular bundle. Pain is triggered by movements of the upper extremity or shoulder and is associated with paresthesias and muscle weakness.